Sunteți pe pagina 1din 60

Selecţie olimpiade şcolare

Clasa a IX a

1. Determinaţi valorile parametrului real m pentru care mulţimea soluţiilor inecuaţiei


min { 2 x -1 , 2 - x } ≤ m este un interval (nevid).
(Romeo Ilie, OL Braşov, 1998)

2. Pentru fiecare pereche ( a, b ) ∈ ℝ × ℝ se defineşte mulţimea A = {a + bm m ∈ ℚ} .


Arătaţi că pentru orice pereche ( a, b ) ∈ ℝ × ℝ avem: { 5, 6, 7 } \ A ≠ ∅
(Valentin Matrosenco, OL Bucureşti)

3. Fie a > 0 . Dacă există cel puţin două valori ale lui x ∈ℚ , x > 0 , astfel încât
a + x + a + x ∈ ℚ , atunci a ∈ ℚ .
(Sorin Rădulescu, Petruş Alexandrescu, OL Bucureşti 2000)

4. Fie M o mulţime de numere reale cu proprietăţile:


(i) ℤ ⊂ M ;
(ii) x, y ∈ M ⇒ x + y ∈ M şi x ⋅ y ∈ M ;
(iii) a = 2 + 3 ∈ M .
1) Găsiţi o ecuaţie cu coeficienţi întregi care are rădăcina a ;
2) Arătaţi că: − 2 + 3 ∈ M
(Gheorghe Eckstein, concurs Traian Lalescu, 1995)

5. Să se determine mulţimile nevide A ⊂ ℝ* care satisfac proprietăţile:


1
i) A are cel mult 5 elemente; ii) x ∈ A ⇒ ∈ A şi 1 − x ∈ A .
x
(Marcel Ţena, concurs GM, 1982)

 mn 1 
6. Pentru fiecare număr natural n ≥ 2 definim mulţimea An =  − n ∈ ℕ  . Arătaţi
n +1 n 
că nu există nici un element comun tuturor mulţimilor An .
(Cristinel Mortici, OJ Argeş 1998)

7. Determinaţi toate mulţimile finite X de numere reale care satisfac:


∀x ∈ X ⇒ x + x ∈ X .
1
8. Arătaţi că dacă x − y ≤ z, ∀z > 0 , atunci x = y .
(Mihai Piticari, OL SV 1984)

9. Arătaţi că dacă numerele a, b, x, y ∈ ℝ satisfac inegalitatea


ax + by ≤ xy , atunci ax + by ≥ 4ab .
(Eugen Păltănea, OL Braşov 1988)

10. Determinaţi toate tripletele ( x, y , z ) de numere întregi pozitive 0 < x ≤ y ≤ z care


1 1 1 4
satisfac: + + = .
x y z 5
* * *, OJ 1988

11. Determinaţi numerele reale b < c < 1 astfel încât pentru orice x ∈ [b, c ] să avem
x
∈ [ b, c ]
x −1
(Constantin Caragea, OL Constanţa, 1989)

12. Fie f : A → A , unde A = {a1 , a2 ,..., an } ⊂ ℕ* , cu a1 < a2 < ... < an , n ≥ 2 , o funcţie cu
1
proprietatea că f ( x ) − f ( y ) ≥ x− y − , ∀x, y ∈ A .
x+ y
Arătaţi că există k ∈ {1, 2,..., n} astfel încât f ( ak ) = a2 .
(Cristinel Mortici, OL Constanţa. 1997)

13. Fie a, b, c ∈ ℕ* astfel încât b > a 2 + c 2 . Arătaţi că ecuaţia ax 2 + bx + c = 0 are


rădăcinile reale şi iraţionale.
(Cristinel Mortici, OL, Constanţa, 1999)

14. Arătaţi că pentru orice n ∈ ℕ* , numărul n − 1 + n + n + 1 este iraţional.


(Liviu Vlaicu, RMT, 1987)

( )( ) ( )
2
15. Rezolvaţi în ℝ ecuaţia x 2 + 2 x + 2 3 x 2 + 2 x + 4 = 3 x 2 + 1 .
(Adriana şi Lucian Dragomir, GM 2001)

( ) + ( 2x ) = ( 3x )
3 3 3
16. Rezolvaţi în ℝ ecuaţia x 2 + 4 x − 5 2
− 7x + 4 2
− 3x − 1 .

2
(Gheorghe Achim, GM 2003)

17. Arătaţi că orice mulţime A de numere naturale consecutive cu proprietatea că


1
∑ a < 2 , conţine cel mult un pătrat perfect.
a∈A
(Cristinel Mortici, GM, 2003)

18. Se consideră mulţimea An = {1, 2,3,..., n} , n ≥ 2 . Notăm cu Sn suma tuturor fracţiilor


subunitare având numitorul şi numărătorul elemente ale lui An .
a) Determinaţi n pentru care Sn ∈ ℕ ;
b) Determinaţi n pentru care Sn = 2475 .
(Marcel Chiriţă, OJ Argeş, 2000)

a 2 + bc b 2 + ca c 2 + ab
19. Arătaţi că pentru orice a, b, c > 0 , are loc: + + ≥ a+b+c.
b+c c+a a+b
(Bogdan Enescu, OJ Bacău, 2000)

20. Determinaţi funcţia f : ℕ → ℕ care satisface simultan proprietăţile:


a) f ( 0 ) = 1 ; b) f ( n ) = f ( n − 1) + 2, ∀n ∈ ℕ* .
(Manuela Prajea, OJ Timiş 2000)

21. Se consideră mulţimea M = {1, 2,3,..., n} . Fie A, B, C submulţimi ale lui M ;


determinaţi numărul tripletelor ( A, B, C ) care satisfac simultan proprietăţile:
a) A ∪ B ∪ C = M ;
b) card ( A ∩ B ∩ C ) = 1 .
(Dorel Miheţ, OJ Bacău 2000)

21. Determinaţi perechile ( x, y ) de numere întregi care verifică:

( ) (
2 x3 − xy + y 3 = 3 x 2 + y 2 . )
(Lucian Dragomir, juriu ON 2002)

23. Pentru orice număr natural n notăm cu p ( n ) cel mai mare pătrat perfect cel mult
 a +1 a + 2
egal cu n . Determinaţi numerele reale a pentru care p  = .
 2  3
(Lucian Dragomir, juriu ON 2002)

3
24. Fie M o mulţime de numere întregi care satisface proprietăţile:
( ) ( )
a) 1∈ M ; b) x ∈ M ⇒ x 2 − x + 1 ∈ M ; c) . x 2 − 3 x + 3 ∈ M ⇒ x ∈ M
Arătaţi că 13 ∈ M .
(Lucian Dragomir, GM 1996)

25. Determinaţi numărul submulţimilor mulţimii A = {1, 2,..., 2n} în care ecuaţia
x + y = 2n + 1 nu are soluţii.
(Olimpiadă Polonia)

26. Determinaţi numerele naturale a şi b care satisfac egalitatea: a ( a + 1) = 4b ( b + 1) .


(Lucian Dragomir, RMT 1998)

27. Fie f : [ 0,1] → ℝ o funcţie cu proprietăţile:


(i) f (1) = 1 ;
(ii) f ( x ) ≥ 0, ∀x ∈ [ 0,1] ;
(iii) Dacă x, y şi x + y sunt din [ 0,1] , atunci f ( x + y ) ≥ f ( x ) + f ( y ) .
Demonstraţi că: f ( x ) ≤ 2 x, ∀x ∈ [ 0,1] .
(Olimpiadă Irlanda)

{ }
28. Fie a, b, c ∈ ℕ* , b > a + c şi A = x ∈ ℝ ax 2 + bx + c = 0 . Demonstraţi că:
a) A are exact două elemente;
b) A ∩ ℤ are cel mult un element;
c) Există a, b, c astfel încât A ∩ ℤ are exact un element.
(Lucian Dragomir, RMT 2004)

29. Fie k ≥ 2 şi A ⊆ ℝ o mulţime cu cel puţin k + 1 elemente şi având proprietatea că


media aritmetică a oricăror k elemente distincte ale sale este tot un element al său.
Demonstraţi că A este infinită.
(Marius Ghergu, RMT 2003)

30. Câte cuvinte distincte, de lungime 1997 pot fi formate utilizând literele A, B, C şi
numai acestea, fiecare de un număr impar de ori ?
(concurs Ungaria, Israel 1997)

31. Determinaţi numărul numerelor naturale n cu următoarele proprietăţi:


a) n are 1000 de cifre; b) toate cifrele lui n sunt impare;
c) modulul diferenţei oricăror două cifre vecine este 2.
(concurs Irlanda)
4
32. Se consideră mulţimile G = {( x, y ) ∈ ℝ 2
}
x 2 − y 2 = 3 x − 4 y şi

H= {( x, y ) ∈ ℝ 2
}
2 xy = 3 x + 4 y .

Să se determine mulţimea M = { z ∈ ℝ z = x 2
}
+ y 2 unde ( x, y ) ∈ G ∩ H .
(Laurenţiu Panaitopol, concurs 1971)

33. Să se determine a, b ∈ ℝ astfel încât funcţia f : [ 0,3] → ℝ ,


 x2 x ∈ [ 0,1)

f ( x ) =  2 x2 x ∈ [1, 2] să aibă proprietatea că orice y ∈ [ 0,8] este imaginea prin f a
ax + b x ∈ ( 2,3]

unei singure valori x ∈ [ 0,3] .
( Ion Cuculescu, concurs 1976)

34. Să se determine numerele reale a şi b astfel încât ecuaţiile x 2 − ax + b = 0 şi


x 2 − bx + a = 0 să aibă rădăcini numere naturale distincte.
(Gheorghe Andrei, OL Constanţa 1993)

35. Determinaţi a, b ∈ ℤ astfel încât ecuaţiile x 2 + ax + 1 = 0 şi x 2 + bx + 2 = 0 să aibă o


rădăcină reală comună.
(Nelu Chichirim, OL Constanţa 2000)

36. Se consideră ecuaţia x 2 − a x + [ a ] ⋅ {a} = 0, a ∈ ℝ .


a) Să se arate că ecuaţia are cel mult 4 soluţii reale şi să se determine valorile lui a
pentru care are exact 4 soluţii, notate cu x1 , x2 , x3 , x4 ;
4
17
b) Determinaţi a pentru care ∑ xk 2 = 2
.
k =1
(Adrian P. Ghioca, OL Prahova, 2000)

1 1 1
37. Să se rezolve ecuaţia: = +
{ x} x [ x ]
(OJ Bucureşti 2000)

38. Să se arate că pentru orice n ∈ ℕ* avem:


 n 2 + n + 1 + n 2 − n + 1  =  4n 2 + 3  .
   
5
(Marian Ursărescu, OJ Buzău 2000)
2
39. Fie a, b, c, d ∈ ℝ astfel încât a + b = c + d şi a + b 2 = c 2 + d 2 . Arătaţi că, pentru
orice n ∈ ℕ , este adevărată egalitatea a n + b n = c n + d n .
(OJ Suceava, 2000)

40. Se consideră mulţimile A = {( x, y ) ∈ ℝ 2


}
x 2 + y 2 > 2 şi

B= {( x, y ) ∈ ℝ 2
x + y + xy < 3 . }
Determinaţi mulţimea A ∪ B (precizând elementele sale).
(Romeo Ilie, OL Braşov, 1998)

41. Determinaţi numerele naturale n pentru care există x natural astfel încât:
1 1 1
+ + ... + =1.
x + x +1 x +1 + x + 2 x + n + x + n +1
(Lucian Dragomir, RMT 1999)

42. Determinaţi funcţiile f : ℕ* → ℕ* cu proprietatea că, pentru orice n ≥ 1 ,


f (1) + f ( 2 ) + ... + f ( n ) este un cub perfect cel mult egal cu n3 .
(Lucian Dragomir, OJ 2003)

43. Să se determine numărul de elemente ale mulţimii


{ }
A = N = a1a2 ...an ai ∈ {2,3,7,9} ,3 \ N , unde n ∈ ℕ, n ≥ 1 este un număr natural nenul
fixat.
(Dorel Miheţ, concurs 2003)

m m 1
44. Fie m, n naturale nenule astfel încât 7− > 0 . Arătaţi că: 7 − > .
n n nm
(Radu Gologan, ON 1978)
45. Arătaţi că dacă [ nx ] = n [ x ] , ∀n ∈ ℕ* , atunci x ∈ ℤ .
(Gheorghe Schneider)
46. Determinaţi x > 0 astfel încât y = x + 2 [ x ] să fie pătratul unui număr
2

întreg.
(Ion Cucurezeanu)
 6 x - 5   3 x + 1   6 x + 15   3 x - 6 
47. Rezolvaţi ecuaţia:  + + +  = 2x + 3
 4   2   4   2 
48. Rezolvaţi ecuaţia: x3 − [ x ] = 3 .
(Concurs Polonia)

6
 2x   2x 
49. Rezolvaţi ecuaţia:  = .
 2 − x   x2 − x + 1
(Const. Caragea, Constanţa 1995)
x  x 
50. Determinaţi y ∈ ℚ pentru care [ x + y ] =   −   , ∀x ∈ ℝ
 y   2y 
(Mircea Lascu, GM)
51.Rezolvaţi ecuaţia: 2 [ x ] = x + { x} .
(Titu Andreescu, RMT)

52. Rezolvaţi ecuaţia: x =


[ x]
{ x}
(Titu Andreescu, RMT)

( )
n
53. Demonstraţi că numărul  2 + 3  este impar, ∀n ∈ ℕ .
 
54. Arătaţi că dacă ( an )n≥1 este o progresie aritmetică cu raţia număr întreg, atunci
şi
bn = [ an ] , n ≥ 1 , este o progresie aritmetică.
(Gheorghe Andrei)
55. Se consideră şirul ( xn )n≥1 definit prin: x1 = x2 = 1 şi xn+1 = n ⋅ xn + xn−1 ,
∀n ≥ 2 . Determinaţi [ xn ] , pentru n ≥ 3 .
(Constantin Caragea)
56. Aflaţi câte numere naturale scrise în baza zece îndeplinesc simultan
următoarele condiţii:
(i) fiecare număr are 6 cifre;
(ii) suma cifrelor fiecărui număr este 9;
(iii) 4 dintre cifrele fiecărui număr sunt 2, 0, 0, 4.
(Lucian Dragomir, OJ 2004)
57. Determinaţi numerele naturale n cu proprietatea că există numerele întregi a
şi b astfel încât: n 2 = a + b şi n3 = a 2 + b 2
(Lucian Dragomir, ON 2004)
58. a) Să se arate că există o infinitate de numere raţionale x > 0 astfel încât
{x } + {x} = 0,99 ;
2

{ }
b) Să se arate că nu există numere raţionale x > 0 astfel încât x 2 + { x} = 1 .
(Bogdan Enescu, OJ 2004)
59. Să se arate că există o infinitate de numere iraţionale a şi b cu proprietatea că
( )
numărul ( a + b ) a 2 + b 2 este pătratul unui număr raţional.
(Alexandru Blaga)
7
60. Fie A o mulţime cu n elemente ( n > 1) . Determinaţi numărul tripletelor
ordonate de mulţimi ( A1 , A2 , A3 ) care satisfac simultan condiţiile:
(1) Ai ≠ ∅,1 ≤ i ≤ 3 ; (2) A1 ∪ A2 = A2 ∪ A3 = A3 ∪ A1 = A ;
(3) A1 ∩ A2 ∩ A3 = ∅ .
(M. Balaj, concurs G.Moisil 1996)
61. Determinaţi funcţiile f : ( 0, ∞ ) → ℝ care satisfac relaţia
f ( xyz ) = xf ( y ) + yf ( z ) + zf ( x ) , ∀x, y , z > 1 .
(Dorel Miheţ, concurs G.Moisil 1997)
62. Să se arate că nu există funcţii strict monotone f : ℝ → ℝ cu proprietatea:
f ( a + b − x ) = f ( a − x ) + f ( x − b ) , ∀x ∈ ℝ , unde a, b ∈ ℝ sunt fixate.
(Lucian Dragomir, OJ Caraş-Severin 1994)
63. Să se determine funcţiile f : ℕ* → ( 0, ∞ ) pentru care avem f ( 4 ) = 4 şi
1 1 1 f ( n)
+ + ... + = , ∀n ∈ ℕ* .
f (1) f (2) f (2) f (3) f (n) f (n + 1) f (n + 1)
(D.M.Bătineţu – Giurgiu, ON 1983)
64. Să se determine funcţiile f : ℚ → ℝ pentru care avem:
f ( x + y ) + f ( x − y ) = 2 f ( x ) + 2 f ( y ) , ∀x, y ∈ ℚ .
65. Determinaţi funcţiile f: ℚ → ℝ care satisfac
f ( x − y ) + f ( y − z ) + f ( z − x ) = 3 ( f ( x ) + f ( y ) + f ( z ) ) − f ( x + y + x ) , ∀x, y, z ∈ ℚ
(Lucian Dragomir, GM 1995)
66. Determinaţi funcţiile f : ℕ → ℕ pentru care avem

( )
f m2 + f ( n ) = f 2 ( m ) + n, ∀m, n ∈ ℕ .
(Lucian Dragomir, OJ 2001)
67. Pentru orice numere naturale a, b notăm E ( a, b ) =  a 2 + a  +  b 2 + b + 1  ,
   
F ( a, b ) =  a 6 + a 3  +  b 6 + b3 + 1  . Determinaţi a, b ∈ ℕ pentru care
   
3 ⋅ E ( a, b ) = 2 ⋅ F ( a, b ) .
(Lucian Dragomir, RMT 2003)
68. Găsiţi funcţiile f : ℝ → ℝ cu proprietatea: f ( x ) + f ([ x ]) + f ({ x}) = 2 x
∀x ∈ ℝ ,
(Dorel Miheţ, RMT 2000)
69. Fie x, y ∈ ℤ astfel încât x + y + ( x + y ) + 30 xy = 2000 . Demonstraţi că
3 3 3

x + y = 10 .
(OBMJ, 2000)

8
70. Într-o sală sunt n matematicieni; fiecare dintre ei cunoaşte exact k
matematicieni. Care este valoarea minimă a lui k pentru a fi siguri că există cel puţin trei
matematicieni astfel încât fiecare să-i cunoască pe ceilalţi doi?
( Short list, OBMJ 2001)
71. La o masă circulară sunt aşezate 7 persoane. Vârsta fiecăreia este media
aritmetică a vârstelor persoanelor alăturate. Arătaţi că suma vârstelor tuturor persoanelor
este multiplu de 7.
(R. Bairac, ON 1994)
72. Arătaţi că numerele 1, 2,3,...,16 nu pot fi aranjate pe o circumferinţă astfel încât
suma oricăror două numere situate pe locuri alăturate să fie pătrat perfect.
(concurs Rusia)
73.La un turneu de tenis au participat de două ori mai mulţi băieţi decât fete.
Fiecare pereche de participanţi a jucat exact o dată (şi nu au fost rezultate egale).
7
Raportul între numărul victoriilor obţinute de fete faţă de cele obţinute de băieţi a fost .
5
Câţi participanţi au fost la acest turneu ?
(OBJ, 2000)
74.Determinaţi funcţiile f :ℝ → ℝ care satisfac relaţia
f ( x + y ) = f 2 ( x ) + f 2 ( y ) , ∀x, y ∈ ℝ
(Marcel Chiriţă, OL Bucureşti 1984)
75. Fie P mulţimea graficelor funcţiilor f : ℝ → ℝ , f ( x ) = ax 2 + bx + c , unde
a, b, c ∈ ℤ, a ≠ 0 , pentru care punctele de intersecţie cu axele formează un triunghi
echilateral. Demonstraţi că P este finită şi determinaţi numărul elementelor sale.
(Alexandru Zaharescu,OL Braşov 1988)
76. Dacă a1 , a2 ,..., a6 ∈ ℝ , a1 < a2 < ... < a6 atunci
( a1 + a2 + ... + a6 )2 > 12(a1a6 + a2 a5 + a3a4 )
(concurs interjudeţean 1988)
( )
1981
77. Demonstraţi că în scrierea numărului 2501 + 50 ca număr zecimal,
primele 3962 cifre după virgulă sunt zerouri.
(D.M.Bătineţu – Giurgiu, ON 1981)
78. Fie A = {( x, y ) ∈ ℝ × ℝ x + y = 2} şi B = {( x, y ) ∈ ℝ × ℝ x 3
}
+ y 3 = 8 − 6 xy .
Determinaţi B \ A
(Dan Mihalca, OL Bucureşti, 1988)
79. Fie mulţimea M = {a + 1, a + 2,..., a + 25} , unde a ∈ ℕ . Să se determine cea
mai mare valoare a lui a astfel încât M să se poată împărţi în trei submulţimi două câte
două disjuncte, cu proprietatea că suma elementelor din fiecare submulţime este cel mult
s+3
, unde s este suma elementelor mulţimii M .
3

9
(Adrian Ghioca, OJ CT 1995)
{
80. Fie mulţimea Ap = x ∈ ℝ x − p + x + 1 = 4 , unde p ∈ ℝ . }
i) Determinaţi valorile parametrului p pentru care Ap este infinită şi determinaţi
Ap ;
ii) Determinaţi p pentru care Ap are un singur element.
(OL Arad 2002)
81. Fie a, b, c ∈ ℝ aşa încât a + b + c = abc = a . Arătaţi că a ≥ 3 .
* 3

(OL Braşov 2002)


n
 1 
82. a) Fie En = ∏  1 + 2  . Să se determine a ∈ ℕ astfel încât inegalitatea
k =1  k 
a
En < 4 − să fie adevărată pentru n = 4 şi să poată fi demonstrată prin inducţie pentru
n
n≥4.
n
 1 
b) Demonstraţi că ∏ 1 + k 3  < 3, ∀n ∈ ℕ* .
k =1
(Laurenţiu Panaitopol, 1995)
83. Să se determine cea mai mare valoare a lui α ∈ ℝ pentru care
1 1 1 α 7
1 + 2 + 2 + ... + 2 + ≤ , ∀n ∈ ℕ, n ≥ 3
2 3 n n 4
(Dorel Miheţ, OJ Timiş 1993)
84. Determinaţi mulţimea {a1 , a2 ,..., an } de numere reale dacă a = 1 şi
( n + 1) an
a1 + a2 + ... + an = , ∀n ∈ ℕ* .
2
(OL Brăila 2002)
85. Determinaţi numerele reale strict pozitive a1 , a2 ,..., an ştiind că a3 = 3 şi
1 1 1 a a
a12 + a22 + a32 + ... + an 2 = n n +1 , ∀n ∈ ℕ* .
2 3 n 2
(Lucian Dragomir, 2002)
86. Fie A o mulţime finită, nevidă, de numere strict pozitive cu proprietatea că:
∀x, y ∈ A ⇒ x + y ∈ A .

( )
a) Arătaţi că a 2 − a ∈ A, ∀a ∈ A b) Determinaţi A .
(Şerban Olteanu, OL Giurgiu 2002)
87. Un elev are 10 bile numerotate cu numerele 1, 2,3,...,10 şi trebuie să le pună în
trei urne identice astfel încât în nici o urnă să nu fie două bile numerotate cu numere
consecutive. În câte moduri se poate face aceasta ?
(juriu ON 2002, clasa a VII-a)

10
88. Câte numere de n cifre, formate numai cu cifrele 1, 9, 8, 6 se divid cu 3 ?
(Dorel Miheţ)
89. Într-un plan se dau 2n + 1 puncte astfel încât oricare trei să fie necoliniare.
Arătaţi că se poate forma cel puţin un triunghi dacă unim cel puţin n 2 + n + 1 perechi de
puncte.
(Mircea Lascu)
90. Arătaţi că ∀n ∈ ℕ, n ≥ 6 , un pătrat poate fi împărţit în n pătrate.
91. Să se determine numărul submulţimilor nevide ale mulţimii {1, 2,...,10} care nu
conţin numere consecutive.
92. Într-o firmă se instalează o staţie internă de telefoane, fiecare număr telefonic
fiind alcătuit din 4 cifre. Directorul firmei cere ca în birourile administrative ale firmei
numerele de telefon să conţină doar cifrele 1, 2, 3 şi oricare două numere din acestea să
aibă cel mult o poziţie în care cifrele ar coincide. Care este numărul maxim de telefoane
ce pot fi instalate în birouri respectând cerinţele directorului ?
(Olimpiadă Moldova, 2002)
93. Fie X ⊂ ℝ o mulţime şi A, B ⊂ X două mulţimi nevide care simultan
proprietăţile:
a) A ∩ B = ∅ şi A ∪ B = X ;
b) ∀x, y ∈ A ⇒ xy ∈ A ;
c) ∀x, y ∈ B ⇒ xy ∈ A ;
d) ∀x,∈ A, ∀y ∈ B ⇒ xy ∈ B
Arătaţi că: 1. Dacă X = ℝ , atunci nu există A, B ⊂ ℝ cu proprietăţile de mai sus;
2. Dacă X = ℝ* , atunci există A, B submulţimi ale lui ℝ cu proprietăţile de mai
sus.
(M.Chiriţă, D. Grigorescu,OL Bucureşti 2001)
94. Fie numerele a1a2 , a3 , a4 , a5 , a6 ∈ ℝ care verifică inegalitatea
( a1 + a2 + a3 + a4 − a5 )2 ≥ 3 ( a12 + a22 + a32 + a42 − a52 ) . Arătaţi că ∀x, y ∈ ℝ are loc

inegalitatea: ( a1 + a2 + a3 + a4 − a5 − x − y ) ≥ a12 + a22 + a32 + a42 − a52 − x 2 − y 2 .


2

(Dorin Andrica, ON 1983)


95. Fie E o mulţime finită şi f : E → E o funcţie cu proprietatea că
(f f )( x ) = x, ∀x ∈ E . Demonstraţi că dacă E are un număr impar de elemente, atunci
există k ∈ E astfel încât f ( k ) = k .
(Gh. Ionescu, OJ 1977)
96. Fie F o mulţime fixată cu n elemente. Determinaţi câte mulţimi E ⊂ ℝ au
proprietatea următoare: pentru f : E → F , f ( x ) = x 2 avem f ( E ) = F .
(Adrian Ghioca, OJ 1985)
97. Se consideră numerele reale a, b, c care satisfac a + b + c = 0 şi
a 2 + b 2 + c 2 = 1 . Demonstraţi că:
11
a) dacă a ≤ b ≤ c atunci b 2 ≤
1
6
{
b) 1 ≤ max ( a − b ) , ( b − c ) , ( c − a )
2 2 2
}.
(Dorel Miheţ, ON 1981)
98. Fie a, b, c ∈ ℤ, a > 0 . Arătaţi că dacă ecuaţia ax + bx + c = 0 admite două 2

rădăcini distincte în intervalul ( 0, 2 ) , atunci a ≥ 2, b ≤ −3 şi c ≥ 1 .


(Mircea Lascu, Liviu Vlaicu, OJ 1986)
99. Fie T = X 2 + aX + b un trinom care admite ca rădăcini numere întregi.. Arătaţi
că dacă c ∈ ℤ astfel încât T ( c ) = 13 , atunci max {T ( c − 1) , T ( c + 1)} = 28 .
(Dorel Miheţ, OJ 1982)
100. Fie X o mulţime finită de numere reale cu proprietatea că: ∀x ∈ X , ∃y ∈ X
astfel încăt x 2 − 2 ≤ y Arătaţi că:
a) −2 ∈ X ⇒ 2 ∈ X b) X ⊂ [ −2, 2]
(Dorel Miheţ, RMT)
101. Dacă a, b, c ≥ 0 , arătaţi că: a + b + c + 3 ≥ 2
2 2 2
( ab + bc + ca . )
(Dinicu Budescu, GM 1998)
102. Arătaţi că: ( ab + bc + ca + a + b + c ) ≥ 4 ( a + b )( b + c )( c + a ) + abc  , oricare
2

ar fi a, b, c ∈ ℝ .
(I. Safta, concurs GM 1999)
103. Dacă numerele reale x, y , z, t au suma egală cu 6, arătaţi că
x2 + y 2 + z 2 + t 2 ≥ 9 .
(I. Safta, concurs GM 1999)
3
x y3 z3 1
104. Fie x, y , z > 0 astfel încât xyz = 1 . Arătaţi că: + + ≥ .
x+ y y+z z+x 2
(Niculai Solomon, GM 1999)
n
1
105. Fie n ∈ ℕ* , x1 , x2 ,..., xn numere reale strict pozitive astfel încât ∑ ≤1.
k =1 1 + xk
n
∏ xk ≥ ( n − 1)
n
Arătaţi că:
k =1
(Dan Ştefan Marinescu, GM 2001)
106. Fie a, b, c numere reale strict pozitive astfel încât a + b + c ≥ abc .
Demonstraţi că: a 2 + b 2 + c 2 ≥ abc 3 .
(Cristinel Mortici, OBM 2001)
107. Arătaţi că pentru orice n ∈ ℕ şi x1 , x2 ,..., xn ∈ [ 0,1] are loc inegalitatea
*

(1 − x1 )(1 − x2 ) ⋅ ... ⋅ (1 − xn ) ≤ 1 − x1x2 ⋅ ... ⋅ xn .


(Şerban Olteanu, OL Giurgiu 2001)
12
1
108. Dacă x, y ∈ ( 0, ∞ ) arătaţi că: x + y + ≥ 3.
xy
(Dinicu Budescu, GM 1998)
3
109. Demonstraţi că: x 2 + xy + y 2 ≥ ( x + y ) , ∀x, y ∈ ℝ .
2
(L.Panaitopol, D.Şt.Marinescu, OL Hunedoara 2002)
3
110. Dacă x, y , z ∈ ( 0,1) , arătaţi că: x(1 − y ) + y (1 − z ) + z (1 − x) ≤ .
2
(Manuela Prajea)
111. Numerele pozitive x1 , x2 , x3 satisfac inegalităţile x1 x2 x3 > 1 ,
1 1 1
+ + > x1 + x2 + x3 . Să se demonstreze că:
x1 x2 x3
a) nici unul din numerele date nu este egal cu 1;
b) exact unul din numerele date este mai mic ca 1.
Autor????????
 1 
112. Arătaţi că  a ∈ ℝ \ {−1,1} a + ∈ ℤ  ⊂ ℝ \ ℚ .
 a 
(Dumitru Buşneag, OL Dolj 1983)
113. Fie a, b, c, a1 , a2 ,..., an ∈ ℝ . Dacă b 2 > ac şi a > n , arătaţi că:
( b + a1 + a2 + ... + an )2 ≥ ( a − n ) ( c − a12 − a22 − ... − an 2 ) .
(Dorin Andrica, concurs Gh.Ţiţeica 1983)
114. Determinaţi numerele naturale nenule n pentru care
1 + 2 + 3 + ... + n  = n
 
(Ionel Tudor, OL Giurgiu 2001)
115. a) Fie x ∈ ℝ astfel încât x + x şi x + 2 x să fie raţionale. Arătaţi că x ∈ℚ ;
2 3

b) Arătaţi că există numere iraţionale x astfel încât x 2 + x şi x3 − 2 x să fie


raţionale.
(Florica Banu, OJ 2002)
116. Arătaţi că pentru orice x ∈ ℝ are loc relaţia:
 x + 3   x + 4   x + 5   x + 1  x + 1
 6 − 6 + 6  =  2 − 3 .
         
(Cristinel Mortici, OJ 2002)
117. Fiind date numerele reale a, c, d , demonstraţi că există cel mult o funcţie
f : ℝ → ℝ astfel încât: f ( ax + c ) + d ≤ x ≤ f ( x + d ) + c , x ∈ ℝ .
(Laurenţiu Panaitopol, ON 2002)

13
118. Determinaţi cel mai mic număr natural n astfel încat să existe n puncte în
plan cu proprietatea că pentru orice alt punct al planului distanţa la cel puţin unul din
aceste puncte este iraţională.
(RMT 1988)
119. Construiţi o funcţie f : ℝ → ℝ care să satisfacă următoarele condiţii:
a) Oricare ar fi două puncte distincte A şi B în plan, segmentul [ AB ] nu este
conţinut în graficul lui f ;
b) f ( f ( x ) ) = 9 x + 1986
(Ştefan Alexe, OL Argeş 1986)
120. Fiecărui număr real x i se asociază numărul t ∈ ℤ care satisface relaţia
2t − x + 2
f ( x ) = t , unde ≤ 0.
2t − x
a) Să se arate că f este funcţie; b) Determinaţi f (1996);
c) Determinaţi x pentru care f ( x ) = 1996 ; d) Reprezentaţi grafic f pentru
x ∈ [ 0, 4] .
(Ilie Stănescu, OJ Sibiu 1996)
121. Determinaţi toate funcţiile f : ℝ → ℝ care satisfac:
f ( x − y ) − xf ( y ) ≤ 1 − x, ∀x, y ∈ ℝ .
(Marcel Chiriţă, OJ Bucureşti 1991)
122. Determinaţi funcţiile f :ℝ → ℝ cu proprietatea că
f ( x + y ) = max { f ( x ) , y} + min { f ( y ) , x} , ∀x, y ∈ ℝ
(concurs T. Lalescu, 1992)
123. Să se determine funcţiile monotone f : ℝ → ℝ pentru care
(f f ... f ) ( x ) = x , ∀x ∈ ℝ .
1995 ori
(concurs T. Lalescu 1995)
124. Fie f : ℕ → ℕ o funcţie cu proprietăţile:
*

a) f ( m ⋅ n ) = f ( m ) + f ( n ) , ∀m, n ∈ ℕ* ;
b) f (10 ) = 0 ;
c) f ( k ) = 0 pentru orice număr k care se termină cu cifra 3.
Determinaţi f (1994 ) .
(concurs T. Lalescu 1994)
125. Fie a, b, c ∈ ℝ, a ≠ 0 , astfel încât b < 4ac . Arătaţi că dacă 4a + 2b + c > 0 ,
2

atunci a + 2b + 4c > 0 .
(Bogdan Enescu, OJ 1995)

14
126. Fie a şi b numere reale strict pozitive, distincte. Considerăm mulţimea:
M = {ax + by x > 0, y > 0, x + y = 1} . Să se demonstreze că:
2ab
a) ∈M ;
a+b
b) ab ∈ M .
(Romeo Ilie, ON 2001)
127. Să se determine numerele reale a şi b ştiind a + b ∈ ℤ şi a 2 + b 2 = 2 .
(Romeo Ilie, ON 2001)
128. Determinaţi toate mulţimile finite A ⊂ ℤ astfel încât ∀x, y ∈ A ⇒ xy ∈ A .
(Dan Popoiu )
129. Se consideră o mulţime formată din 2001 puncte în plan. Să se arate că există
un cerc care trece printr-un singur punct din mulţimea A şi care conţine în interior exact
1000 de puncte din mulţimea A .
(Marian Andronache, Ion Savu, 2001)
130. Pentru ce valori ale numărului real a mulţimea [ 0, a ] ∪ [1, 2] este interval ?
(concurs Gh. Mihoc, 2003)
131. Să se determine m ∈ ℝ pentru care x 2 − 2mx + m ≥ 0, ∀x ∈ [ −1,1] .
(Dorel Miheţ, concurs interjudeţean 1986)
132. Să se determine m ∈ ℝ pentru care x 2 − mx + 2 ≥ 0, ∀x ∈ ℤ .
(Jenică Crînganu, OL Galaţi 1990)
133. Fie f : ℕ → ℕ o funcţie crescătoare cu proprietatea că există x ∈ ℕ astfel
încât f ( x ) < x . Să se arate că există y ∈ ℕ astfel încât f ( y ) = y .
(Marius Gârjoabă, OJ Sibiu 1994)
134. Fie f un polinom de grad doi, cu coeficienţi întregi, pentru care există
u, v ∈ ℤ , u ≠ v , astfel încât f ( u ) = v şi f ( v ) = u . Demonstraţi că ecuaţia f ( x ) = x are
rădăcini iraţionale.
(Laurenţiu Panaitopol, OJ Bucureşti, 1991)
135. Să se determine n ∈ ℕ* astfel încât x − 1 + x − 2 + ... + x − n < x ,
∀x ≥ n .
(Laurenţiu Panaitopol, concurs GM 1996)
136. Să se determine toate polinoamele f de gradul doi cu coeficienţi întregi
pentru care numerele f ( 2 ) , f ( 5 ) şi f ( 8 ) au partea întreagă 0.
(Laurenţiu Panaitopol, OJ Bucureşti, 1994)
137. Fie Pm familia de parabole de ecuaţii y = x 2 − mx − 1 , unde m este un
parametru real. Notăm Pm ∩ Ox = { Am , Bm } şi Pm ∩ Oy = {Cm } . Determinaţi locul
geometric al centrului cercului circumscris triunghiului Am BmCm .
(Dorel Miheţ, OJ Timiş, 1991)
15
138. Fie A o mulţime de numere reale care satisface simultan proprietăţile:
a) 1∈ A ; b) x ∈ A ⇒ x 2 ∈ A ; c) x 2 − 4 x + 4 ∈ A ⇒ x ∈ A .
Arătaţi că 2000 + 2001 ∈ A
(Lucian Dragomir, ON 2001)
139. Fie a, b ∈ ℕ* cu a < b şi C ⊆ [ a, b ] ∩ ℕ* astfel încât numărul elementelor lui
b − a +1
C să fie strict mai mare decât . Să se arate că există două elemente din C care
2
au suma a + b .
(Radu Miculescu, concurs 2003)
140. Calculaţi minimul expresiei
E ( a, b, c ) = max (1 − a, b + c ) + max (1 − b, c + a ) + max (1 − c, a + b ) , unde a, b, c ∈ ℝ .
(Laurenţiu Panaitopol, concurs 2003)
141. Se consideră triunghiul ABC cu lungimile laturilor AB = c , BC = a , CA = b
şi se notează cu P intersecţia dintre mediana BD , D ∈ AC şi bisectoarea ( CE a
unghiului ∢BCA, E ∈ AB . Determinaţi, în funcţie de a, b, c , numerele reale x şi y
pentru care avem PA = xPB + yPC .
(OL Bihor 2001)
142. Fie ABC un triunghi ascuţitunghic cu ortocentrul H . Dacă pentru orice
punct M din planul triunghiului există relaţia MA + MB + MC = 3MH , demonstraţi că
ABC este triunghi echilateral.
(D.M.Bătineţu – Giurgiu, GM 2000)
143. În patrulaterul convex ABCD se notează cu G centrul de greutate al
triunghiului BCD şi cu H ortocentrul triunghiului ACD . Să se arate că punctele
A, B, G , H reprezintă,în această ordine, vârfurile unui paralelogram dacă şi numai dacă
G este centrul cercului circumscris triunghiului ACD .
(Marian Andronache, OL Bucureşti 2001)
144. Fie triunghiurile ABC şi A1B1C1 având ortocentrele H şi H1 , iar O şi O1
fiind centrele cercurilor circumscrise celor două triunghiuri. Arătaţi că dacă
HH1 + 2OO1 = 0 , atunci cele două triunghiuri au acelaşi centru de greutate.
(Gabriela Constantinescu, OL Constanţa, 2001)
145. Se dă un pătrat cu lungimea laturii 1.
a) Să se arate că oricum am considera cinci puncte în interiorul său, există cel puţin
2
două astfel încât distanţa dintre ele să fie mai mică decât ;
2
b) se poate găsi o propoziţie analoagă pentru hexagonul regulat de latură 1?
(OJ 1972)

16
146. Fie ABCD un patrulater convex cu M , P ∈ ( AB ) şi N , Q ∈ ( CD ) astfel încât
AM DQ BP CN
= = = = k . Să se demonstreze că:
MB QC PA ND
a) MQ + PN = AD + BC ; b) MN + PQ = AC + BD .
(Gh.Andrei, 2003)
147. Se consideră dreptunghiul ABCD în care AB = a şi BC = b . a) Determinaţi
locul geometric al punctelor G din plan pentru care mGA − GB + GC = 0 , m ∈ ℝ* ;
b) Determinaţi locul geometric al punctelor M din plan pentru care
MA − MB + MC = a 2 + b 2 .
(Daniela Burtoiu, OL Argeş 2002)
148. În triunghiul ABC considerăm centrul I al cercului înscris şi punctele
AM AN
M ∈ ( AB ) , N ∈ ( AC ) . Notăm m = şi n = . Demonstraţi că punctele M , I , N
AB AC
b c
sunt coliniare dacă şi numai dacă + = a + b + c (notaţiile sunt cele uzuale)
m n
(Romeo Ilie, OL Braşov 2002)
149. Arătaţi că dacă a ⋅ OA + b ⋅ OB + c ⋅ OC = (a + b + c) ⋅ OG atunci triunghiul
ABC este echilateral (notaţiile sunt cele cunoscute).
(OL Brăila 2002)
150. Fie ABCD un patrulater inscriptibil, iar M , N , P, Q mijloacele laturilor
( AB ) ,
( BC ) , ( CD ) , ( DA) . Arătaţi că perpendicularele din M pe CD , din N pe DA , din
P pe AB şi din Q pe BC sunt concurente.
(concurs Traian Lalescu 2003)
151. Fie ABCD un patrulater convex, M ∈ ( BC ) , N ∈ ( CD ) şi
BP BM AP
{P} = ( AM ) ∩ ( BN ) . Demonstraţi că dacă = ⋅ , atunci patrulaterul dat are
BN BC AM
două laturi paralele.
(Dan Ştefan Marinescu, Ioan Şerdean, OL HD 2001)
152. Fie ABC un triunghi. Folosind notaţiile uzuale, arătaţi că IG BC dacă şi
numai dacă AB + AC = 2 BC .
(OL Vaslui 2001)
153. În triunghiul ABC notăm cu D, E , F punctele în care bisectoarele interioare
ale unghiurilor ∢A, ∢B, ∢C intersectează laturile triunghiului. Arătaţi că dacă
AD, BE , CF formează un triunghi, atunci ∆ABC este echilateral.
(OL Vrancea 2001)

17
154. Considerând în plan doi vectori u şi v , demonstraţi echivalenţa afirmaţiilor
următoare: a) u = v ; b) au + bv = bu + av , ∀a, b ∈ ℝ .
(Marcel Ţena, OL Bucureşti 2001)
155. Se consideră rombul ABCD şi punctele M ∈ ( AB ) , N ∈ ( BC ) , P ∈ ( CD ) . Să
se arate că centrul de greutate al triunghiului MNP aparţine dreptei AC dacă şi numai
dacă AM + DP = BN .
(Marian Andronache, OL Bucureşti, 2002)
156. Se dă triunghiul ABC şi punctele L, M astfel încât AL = n AB + AC , iar
n
CM = CB . Să se demonstreze că punctele A, M şi L sunt coliniare.
n +1
(Ioan Cuc, OL Bihor 2003)
157. Se consideră punctele A, B, C , D coplanare, oricare trei necoliniare şi R, S
ortocentrele triunghiurilor ABC , respectiv ABD . Să se arate că A, B, C , D sunt
conciclice dacă şi numai dacă RS = CD .
(M. Andronache, OL Bucureşti, 2003)
158. Fie triunghiul ABC , punctul M ∈ ( BC ) şi cercurile C0 = C ( I , r ) ,
C1 = C ( I1 , r1 ) şi C2 = C ( I 2 , r2 ) cercurile înscrise în triunghiurile ABC , ABM ,
respectiv ACM . Să se arate că: a) cercurile C1 şi C2 sunt tangente dacă şi numai dacă
M ∈ C0 ;
b) dacă M ∈ C0 , atunci există relaţia: p AI = a AS + ( p − a) AD , unde S , D sunt
IS p − a
mijloacele segmentelor ( AM ) , respectiv ( BC ) , I ∈ SD , =
ID a
(Virgil Nicula, OL Bucureşti 2003)
159. Fie ABCD un paralelogram şi M ∈ ( AD ) , N ∈ ( AB ) , P ∈ ( BC ) astfel încât
AD
MD + PC = ⋅ NB . Să se arate că centrul de greutate al triunghiului MNP se găseşte
AB
pe diagonala AC .
(Cătălin Zârnă, OL Constanţa, 2003)
160. Fie X un punct în interiorul triunghiului ABC , M , N ∈ BC , P, R ∈ CA ,
Q, S ∈ AB astfel încât MR AB, SP BC , NQ CA, MR ∩ SP ∩ NQ = { X } , iar A1 , B1 , C1
mijloacele segmentelor ( MN ) , ( PR ) , respectiv ( QS ) . Arătaţi că:
3
a) XA1 + XB1 + XC1 = XG ;
2
3
b) A1 A + B1B + C1C = XG , unde G este centrul de greutate al ∆ABC .
2
(Ovidiu Pop, OL Satu – Mare 2003)

18
161. Fie triunghiul ABC în care O este centrul cercului circumscris, H
ortocentrul şi G centrul de greutate. Pe semidreptele ( OA , ( OB , ( OC se consideră
OD OE OF
punctele D, E , F astfel încât = = =k , k >2 şi pe segmentele
OA OB OC
DM EN FP
( DB ) , ( EC ) , ( AF ) se consideră punctele M , N , P astfel ca = = = k − 2 . Să
MB NC PA
se arate că centrul de greutate al ∆MNP este mijlocul lui ( HG ) .
(Cătălin Zârnă, OL CT 2002)
162. (i) Fie u şi v doi vectori în plan şi a ∈ [ 0,1] . Arătaţi că
u + v ≤ au + (1 − a )v + (1 − a )u + av ≤ u + v ;
(ii) Dacă ABC este un triunghi, D mijlocul lui ( BC ) , M , N ∈ ( BC ) cu
( BM ) ≡ ( CN ) , demonstraţi că: 2 AD ≤ AM + AN ≤ AB + AC .
( Marius Cavachi, Dan Ştefan Marinescu, OL HD 2002)
163. Fie ABC un triunghi înscris în cercul de centru O . Dacă M , N , P sunt
simetricele lui O faţă de BC , CA respectiv AB , arătaţi că dreptele AM , BN , CP sunt
concurente.
(E. Morariu OL Neamţ 2002)
164. În paralelogramul ABCD avem AB = 4, BD = 3, BC = 2 . Fie G centrul de
greutate al ∆ABD , I centrul cercului înscris în ∆BCD şi M ∈ ( BC ) astfel încât
BM = 2MC . Să se demonstreze că:
4 PB + 3PC + 2 PD
a) PI = , oricare ar fi punctul P din planul paralelogramului ;
9
b) punctele G , I şi M sunt coliniare.
(Olosz Ferenc, OL Satu Mare, 2002)
165. Fie ABCD un patrulater inscriptibil şi M un punct pe cercul circumscris
acestuia, diferit de vârfurile patrulaterului. Fie H1 , H 2 , H 3 , H 4 ortocentrele triunghiurilor
MAB, MBC , MCD , respectiv MDA iar E şi F mijloacele segmentelor ( AB ) , respectiv
( CD ) . Demonstraţi că: a) H1H 2 H 3 H 4 este paralelogram; b) H1H 3 = 2 EF .
(Nicolaie Muşuroia, OJ 2002)
166. Fie ABC un triunghi, G centrul său de greutate şi punctele M ∈ ( AB ) ,
AM BN CP
N ∈ ( BC ) , P ∈ ( CA ) astfel încât = = . Notăm cu D, E , F centrele de
MB NC PA
greutate ale triunghiurilor AMP, BMN , CNP . Demonstraţi că;
a) ∆ABC şi ∆DEF au acelaşi centru de greutate;
b) pentru orice punct X din planul ∆ABC avem:
3XG < XD + XE + XF < XA + XB + XC .
(Dan Ştefan Marinescu, Viorel Cornea, OJ 2002)
167. Fie O centrul cercului circumscris triunghiului ABC .
19
Dacă OA + 2OB = OB + 2OC = OC + 2OA , atunci triunghiul ABC este
echilateral.
(Liviu Ignat, concurs Gh.Dumitrescu, 2002)
168. Fie A o submulţime de vectori din plan cu proprietăţile:
a) A conţine orice vector de lungime 1;
b) pentru u, v ∈ A avem u + v ∈ A .
Arătaţi că A conţine toţi vectorii din plan.
(concurs Radu Miron 2001)
169. Fie un triunghi ABC . Arătaţi că pentru orice M ∈ ( AB ) , N ∈ ( AC ) , avem:
BC ⋅ MN ≥ AB ⋅ AM + AC ⋅ AN dacă şi numai dacă m ( ∡A ) ≥ 90 .
(Laurenţiu Panaitopol, GM 1999)
170. Fie triunghiul ABC şi G un punct în interiorul său cu proprietatea că există
un punct M în planul său astfel încât 3MG = MA + MB + MC . Să se arate că G este
centrul de greutate al triunghiului ABC .
(Dan Ştefan Marinescu, Viorel Cornea, 2001)
171. Fie AM , BN , CP lungimile unei înălţimi, a unei bisectoare şi a unei mediane
(în această ordine) în triunghiul ascuţitunghic ABC . Să se demonstreze că dacă
MC = NA = PB , atunci ABC este echilateral.
(Viorel Băndilă, OL Bucureşti 1987)
172. Fie ( AD , ( BE , ( CF bisectoarele interioare ale unghiurilor triunghiului ABC .
Arătaţi că dacă DB = EC = FA , atunci triunghiul ABC este echilateral.
(Emil Constantinescu, OL Bucureşti 1990)
173. Fie ABCD un trapez cu AB CD , iar M şi N mijloacele segmentelor
( AB ) şi respectiv ( CD ) . Considerăm E ∈ ( AD ) diferit de mijlocul lui ( AD ) . Paralela
prin E la baze taie pe ( BC ) în F . Să se arate că dreptele MF , NE , AC sunt concurente.
(Constantin Cocea, OL Iaşi 1990)
174. Fie ABC un triunghi, M şi N mijloacele laturilor ( BC ) , respectiv ( AC ) şi
P ∈ ( AB ) astfel încât PA = 2 PB . Fie {G} = BN ∩ AM şi {Q} = BN ∩ CP . Dacă
BM ⋅ BP = BQ ⋅ BG , arătaţi că AB este perpendiculară pe BC dacă şi numai dacă
AB = BC .
(D.M.Bătineţu – Giurgiu, OJ 1986)
175. Fie ABC un triunghi oarecare. Pe semidreptele ( AB şi ( AC se consideră
2AB ⋅ AC
punctele E , respectiv D , aşa încât AE = AD = . Arătaţi că DE , BC şi
AB + AC
bisectoarea interioară a unghiului A sunt concurente.
(OJ 1986)

20
176. În triunghiul ABC se consideră bisectoarele ( BD şi ( CE , cu D ∈ ( AC ) ,
E ∈ ( AB ) . Fie M şi N mijloacele segmentelor ( BD ) , respectiv ( CE ) .
Să se arate că dreptele EM , DN , BC sunt concurente dacă şi numai dacă
m ( ∢A ) = 60 .
(Laurenţiu Panaitopol, OJ 1988)
177. Fie ABC un triunghi, D ∈ ( BC ) şi CD = k ⋅ BC .
a) Să se demonstreze că AD < k ⋅ AB + (1 − k ) ⋅ AC ;
b) Dacă ( AD este bisectoarea interioară a unghiului A , arătaţi că:
2 1 1
> + .
AD AB AC
(Titu Andreescu, ON 1983)
178. Se consideră un triunghi oarecare ABC şi cercurile C1 ( A, r1 ) , C2 ( B, r2 )
C3 ( C , r3 ) tangente două câte două. Dacă C1 ∩ C2 = {C1} , C2 ∩ C3 = { A1}
C3 ∩ C1 = {B1} , arătaţi că:
a) AA1 , BB1 , CC1 sunt concurente;
b) Dacă punctul de intersecţie al dreptelor de la punctul a) este centrul cercului
înscris în ∆ABC atunci acest triunghi este echilateral.
(Lucian Dragomir, RMT 1999)
179. Pe laturile ( AB ) şi ( AC ) ale triunghiului ABC se consideră punctele M şi
respectiv N astfel încât BM = CN . Arătaţi că dreapta care uneşte mijloacele
segmentelor ( MN ) şi ( BC ) este paralelă cu bisectoarea unghiului ∡BAC .
***
180. Fie ABC un triunghi, P mijlocul lui ( BC ) şi D ∈ ( AB ) , E ∈ ( AC ) astfel
încât DE BC . Semidreptele ( CD şi ( BE intersectează paralela prin A la BC în G ,
respectiv F . Dacă M şi N sunt respectiv mijloacele segmentelor ( AG ) şi ( AF ) ,
demonstraţi că dreptele BN şi CM sunt concurente.
(Lucian Dragomir, RMT 2000)
181. Fie ABCD un patrulater convex şi
AB = a, BC = b, CD = c, DA = d , AC = e, BD = f .

( )
a) Arătaţi că: 2 e 2 + f 2 ≤ ( a + c ) + ( b + d ) ;
2 2

b) Dacă ABCD este circumscriptibil, atunci 2ef ≤ ab + bc + cd + da .


(Dan Ştefan Marinescu, OJ Hunedoara, 1995)

21
182. Pe laturile ( BC ) şi ( CD ) ale patrulaterului convex ABCD se consideră
BM CN
respectiv punctele M şi N astfel încât = 2 şi = 3 . Fie AM ∩ BN = {P} astfel
MC ND
AP BP 4
încât = 2 şi = . Arătaţi că ABCD este paralelogram.
PM PN 5
(Maria Elena Panaitopol, RMT 2000)
183. Stabiliţi natura triunghiului ale cărui laturi verifică:
(a + b ) ( )
2
2 +c 3 = 6 a 2 + b2 + c 2
(Ion Cheşcă, OJ CL 1993)
184. Arătaţi că raportul dintre cea mai mare diagonală şi cea mai mică latură a unui
1+ 5
pentagon convex este mai mare sau egal cu .
2
(Florin Vulpescu – Jalea, OL Bucureşti, 1991)
185. Fie I punctul de concurenţă al bisectoarelor interioare AD, BE , CF ale
IA IB IC
unghiurilor triunghiului ABC şi = x, = y, = z . Demonstraţi că:
ID IE IF
xy + yz + zx ≥ 2 ( x + y + z ) ≥ 12 .
(Marcel Chiriţă, OL Bucureşti 1995)
186. Să se arate că ∆ABC unde A, B, C A, B, C ∈ C ( 0,1) este echilateral dacă şi
numai dacă
2 2 2
OA + OB + OB + OC + OC + OA = 3 .
(Costică Grigoriu, OL Neamţ, 2002)
187. Fie M o mulţime de numere reale cu proprietăţile:
1) 0 ∈ M ;
2) x ∈ M ⇒ ( sin x + cos x ) ∈ M ;
3) ( sin 2 x + cos 2 x ) ∈ M ⇒ x ∈ M .
Să se arate că:

a) ∈ M ; b) M conţine o infinitate de numere iraţionale subunitare.
4
(Lucian Dragomir, RMT, 2002)
 x0 = tg 220
188. Arătaţi că o soluţie a ecuaţiei x + xy + y = 1 este dată de 
 y0 = tg 23
0

 π π
189. Fie x, y , z ∈  − ,  astfel încât
 2 2

22
cos 2 ( x − y ) + cos 2 ( y − z ) + cos 2 ( z − x ) = 1 . Să se demonstreze că două dintre
π
numerele x, y, z au diferenţa egală cu .
2
(Iaroslav Chebici, OL Buc. 1996)
3 + cos 2 A
190. Arătaţi că în orice triunghi ABC avem: > cos 2 B + cos 2 C .
2
(ON 1977)
191. Arătaţi că în orice triunghi ABC are loc relaţia:
A− B C
sin 2 + sin A ⋅ sin B + sin 2 = 1 .
2 2
(ON 1980)
192. Fie A ⊂ ( 0,1] o mulţime cu patru elemente. Arătaţi că există x, y ∈ A astfel
1
încât 0 < x ⋅ 1 − y 2 − y ⋅ 1 − x 2 < .
2
(Dan Ştefan Marinescu, OJ Hunedoara, 1994)
193. Fie ABC un triunghi ascuţitunghic cu latura cea mai mică ( BC ) . Înălţimea
din A intersectează, a doua oară, cercul circumscris în D . Să se arate că:
a) AD ≥ BH + CH unde H este ortocentrul ∆ ABC ;
b) cos ( B − C ) ≥ cos B + cos C .
(Liliana Niculescu, Dan Ştefan Marinescu, OJ Hunedoara 1994)
194. Să se arate că în orice triunghi ABC este adevărată egalitatea
A B C A B C
ctg + ctg + ctg = ctg ctg ctg .
2 2 2 2 2 2
Utilizând eventual această egalitate, demonstraţi inegalitatea:
A B C A− B B−C C−A
8sin sin sin ≤ cos cos cos .
2 2 2 2 2 2
(I.V. Maftei, OJ 1984)
195. Să se demonstreze că pentru orice triunghi ABC are loc inegalitatea:
R (a + b)(b + c)(c + a )
≥ .
r 4abc
Să se deducă de aici inegalitatea lui Euler ( R ≥ 2r ).
(Dorin Andrica, OJ 1985)
196. Fie D, E , F mijloacele arcelor mici BC , CA respectiv AB ale cercului
circumscris triunghiului ascuţitunghic ABC . Demonstraţi că dacă triunghiurile BDC ,
CEA şi AFB au ariile egale, atunci triunghiul ABC este echilateral .
(Lucian Dragomir, RMT 2003)
197. Să se determine numerele n ∈ ℕ ştiind că cos nx ≤ n ⋅ cos x , ∀x ∈ ℝ .
*

(Gheorghe Iurea, concurs 2002)

23
198. Se consideră pentagoanele convexe ABCDE înscrise într-un cerc de rază 1,
care au diagonalele ( AC ) şi ( BD ) perpendiculare. Să se determine valoarea maximă a
ariilor acestor pentagoane.
(ON 1988)
 π n
199. Dacă x1 , x2 ,..., xn ∈ ℝ , a ∈  0,  şi ∑ sin xk ≥ n sin a , atunci
 2 k =1
n
∑ sin( xk − a) ≥ 0 .
k =1
(Sorin Rădulescu, baraj 1983)
200. Fie x, y ∈ ℝ . Arătaţi că dacă mulţimea An = {cos nπ x + cos nπ y n ∈ ℕ} este
finită, atunci x ∈ℚ şi y ∈ ℚ .
(Vasile Pop, baraj 1996)
201. Să se rezolve în ℝ ecuaţia { x} − {2 x} = x .
(Costel Chiteş, Ol. Bucureşti, 2000)
202. Să se arate că dacă a, b, c > 0 şi ab + bc + ca = 1 , atunci
1 1 1
+ + ≥ 3( a + b + c ) .
a b c
(Valentin Vornicu, Ol Bucureşti, 2004)
203. Rezolvaţi în ℕ* inecuaţia
(1 − xy ) (1 − x3 )(1 − y3 ) ≤ 3 (1 − x )(1 − y ) (1 − x3 y3 ) .
(Ovidiu Bădescu, Ol. Caraş Severin,
2004)
204. Dacă x, y, x sunt numere strict pozitive, cu x + y + z = 1 , arătaţi că
x + yz + y + zx + z + xy ≤ 2 .
(Tudorel Lupu, Ol. Constanţa, 2004)
205. Să se determine toate mulţimile finite A de numere naturale care au
proprietatea: ∀x, y ∈ A ⇒ ( xy − x ) ∈ A .
(Ol. Dolj, 2004)
206. Să se determine funcţiile strit crescătoare f : {1,2,...,10} → {1, 2,...,10} care
au proprietatea că ( x + y ) divide ( x ⋅ f ( x ) + y ⋅ f ( y ) ) , ∀x, y ∈ {1,2,...,10} .
(ON 2004)
207. Dacă a, b > 0 şi a + b = 1 determinaţi minimul expresiei
1 1
E= + .
1− a 1− b
(Shorlist, ON 2004)
208. Dacă a, b, c ∈ ℝ şi a + b + c = 3 , arătaţi că a + b + c − abc ≤ 4 .
2 2 2

24
(V. Nicula, OJ, 2004)
209. a) Arătaţi că dacă a, b, c ∈ ℝ şi a + b + c = 6 , atunci a + b 2 + c 2 ≥ 12 .
2

b) Rezolvaţi în ℝ × ℝ ecuaţia
x − y + 3 + 3y − 2x + 4 + x − 2 y + 5 = 6 .
(Petrişor Neagoe, OL Bihor, 2006)
210. Determinaţi numerele raţionale x şi y pentru care există m, n ∈ ℕ astfel
1 1
încât x + y = m şi + = n .
x y
(Nicolae Dragomir, Tudor Diaconu, OL Bihor, 2006)
211. Pentru orice funcţie f : ℝ → ℝ şi orice număr natural m notăm
{ }
Am = x ∈ ℝ f ( x ) = m . Spunem că o funcţie f : ℝ → ℝ este o funcţie „simplă” dacă
pentru orice m ∈ ℕ , mulţimea Am are cel mult două elemente.
a) Demonstraţi că nu există funcţii „simple” f cu proprietatea că
xf ( x ) + f ( − x ) = x + 1, ∀x ∈ ℝ .
b) Determinaţi funcţiile „simple” f care satisfac: xf ( x ) + (1 − x ) f ( − x ) = x + 1, ∀x ∈ ℝ .
(Lucian Dragomir, OL Bihor, 2006)
212. Pe laturile ( AB ) , ( BC ) , ( CA ) ale triunghiului ABC se consideră punctele
M , N , P astfel încât AM = BN = CP . Demonstraţi că dacă triughiurile ABC şi MNP au
acelaşi centru de greutate, atunci tringhiul ABC este echilateral..
(Nicolae Stăniloiu, OL. Bihor, 2006)
1 + xy 1 + yz 1 + xz
213. Dacă x, y , z ∈ ( 0, ∞ ) şi xyz = 1 , atătaţi că + + ≥ 3.
1+ z 1+ x 1+ y
(OL. Iaşi, 2006)
214. Fie x, y , z ∈ ( 0, ∞ ) . Să se demonstreze inegalitatea
1 1 1 1 1 1 1 
+ + ≤  + + .
x + yz
2
y + zx
2
z + xy 2  xy yz zx 
2

(Traian Tămâian, OJ 2006)


215. Pentru fiecare n ∈ ℕ, n ≥ 2 notam cu p ( n ) cel mai mare număr prim mai
mic sau egal cu n şi q ( n ) cel mai mic număr prim mai mare strict ca n . Arătaţi că
n
1 1
∑ p(k )q(k ) < 2 .
k =1
(Nicolae Stăniloiu, OJ. 2006)

( )
2004 
216. Arătaţi că numărul a = 1 +  13 + 3  se divide prin 22004 .
 
(Mihai Chiş, Concurs Traian Lalescu 2004)
217. Să se determine funcţiile f : ℕ → ℕ care satisfac :

25
a) 0 ∈ f ( ℕ ) ;
b) f ( f ( n − 1) + 1) = n, ∀n ∈ ℕ* .
(Dan Comănescu,Concurs Traian Lalescu 2004)
218. Cele 2n pătrăţele ale unui dreptunghi de dimensiuni 2 × n se colorează cu
trei culori. Spunem că o culoare are o „tăietură” dacă pe una din cele n coloane
avem două pătrate de aceeaşi culoare. Să se determine:
a) numărul colorărilor fără „tăieturi”;
b) numărul colorărilor cu o singură „tăietură”.
(Daniela Inoan, OJ 2007)
219. Se consideră triunghiul ABC şi punctele
M ∈ ( AB ) , N ∈ ( BC ) , P ∈ ( CA ) , R ∈ ( MN ) , S ∈ ( NP ) , T ∈ ( PM ) astfel încât
AM BN CP MR NS PT
= = = α şi = = = 1 − α cu α ∈ ( 0,1) .
MB NC PA RN SP TM
a) Să se arate că ∆STR ~ ∆ABC ;
b) Să se determine valoarea parametrului α pentru care aria triunghiului STR este
minimă.
(Marian Teler, OJ. 2007)
220. Să se determine funcţiile f : ℕ* → ℕ* pentru care x 2 + f ( y ) divide
f 2 ( x ) + y pentru orice x, y ∈ ℕ* .
(Lucian Dragomir, OJ 2007)

221. Se considerǎ funcţia f : ℤ → ℚ care satisface simultan urmǎtoarele proprietǎţi:


a) f ( x − y ) = f ( x) − f ( y ) , ∀x, y ∈ ℤ ;
b) f (3k ) = 3k −1 , ∀k ∈ ℕ .
Calculaţi f (2007) .
Lucian Dragomir, GM, Concurs RMCS, 2007, clasa a VIII a
222. Studiaţi dacǎ existǎ funcţii strict monotone f : ℝ → ℝ care satisfac

( ) 1
f 3x − f 2 ( 2 x + 1) ≥ , ∀x ∈ ℝ .
4
Concurs RMCS, 2007
{ } {
223. Se considerǎ mulţimile A = x ∈ ℤ / x 2 + 3 x − m = 0 şi B = x ∈ ℤ / x 2 − 4 x + m = 0 . }
Determinaţi m ∈ ℝ ştiind cǎ existǎ a, b ∈ A ∪ B astfel încât a + b = 3.
Lucian Dragomir, Concurs RMCS, 2008

26
224. Se noteazǎ cu M mijlocul laturii (BC) a unui triunghi ascuţitunghic ABC, iar
π
proiecţiile lui M pe AB şi AC se noteazǎ cu P, respectiv Q. Arǎtaţi cǎ A = dacǎ şi
6
numai dacǎ 4 ⋅ ( MP + MQ ) = AB + AC .
Concurs RMCS, 2008
1 1 1
225. Determinaţi numerele naturale nenule a, b, c pentru care + + ≥ 1 şi
a b c
a⋅ 3 +b
=2.
b⋅ 3 +c
Lucian Dragomir, Concurs RMCS, 2009, clasa a VII a
226. Demonstraţi cǎ, dacǎ x, y ∈ [1;3] şi x ⋅ y = 4 , atunci
x y
2≤ + ≤ 2⋅ 3 .
4 y − y2 4x − x2
Ovidiu Bǎdescu, Concurs RMCS, 2009
227. Numerele 9, 25 şi 49 sunt termeni ai unei progresii aritmetice cu raţia strict pozitivǎ.
Demonstraţi cǎ numǎrul 2009 este deasemenea termen al acestei progresii.
Concurs RMCS, 2009
228. Determinaţi funcţiile f : ℕ → ℕ care satisfac :
a) f (n + m) = f (n) + f (m) + 2mn , ∀m, n ∈ ℕ ;
b) f (n) este pǎtrat perfect, pentru orice n ∈ ℕ .
Marcel Chiriţǎ, Marian Andronache, OL Bucureşti
1 1 1 3
229. Arǎtaţi cǎ, dacǎ a, b, c > 0 şi abc = 1 , atunci 2 + 2 + 2 ≥ .
a (b + c) b (c + a ) c (a + b) 2
Daniel Drimbe, Marius Cicortaş, OL Bihor, 2008
230. Dacǎ A ⊂ ℝ este o mulţime cu cel puţin trei elemente şi care are proprietatea cǎ,
pentru orice douǎ elemente distincte x, y ∈ A , avem şi ( x + y ) ∈ ℚ , demonstraţi cǎ
A⊂ℚ.
OL Bucureşti, 2008
ab 1  1
231. Arǎtaţi cǎ, dacǎ a, b, c > 0 şi a + b + c = 1 , atunci ∑ ≤ ⋅3 + ∑  .
1− c 8  a
OL Iaşi, 2008
232. Se considerǎ mulţimea A = {1, 2,3,...,98} . Arǎtaţi cǎ, oricum am alege 50 de
elemente ale mulţimii considerate, existǎ douǎ printre ele care au suma cub perfect.
Gabriel Popa, OL Iaşi, 2008
233. a) Arǎtaţi cǎ, dacǎ x, y > 0 şi xy = 1 , atunci 4 + x + y ≥ 3( x + y ) ;
2 2

b) Dacǎ a, b, c, d > 0 satisfac abcd = 1 , arǎtaţi cǎ:


( )( )
8 + a 2 + b 2 c 2 + d 2 ≥ 3(a + b)(c + d ) .

27
Andrei Eckstein, Concurs T.Lalescu, 2008
b + kc c + ka a + kb
234. Fie k ∈ ℝ fixat şi numerele reale nenule a, b, c pentru care = = .
a b c

Determinaţi mulţimea valorilor expresiei E =


( a + b + c )3 .
abc
Andrei Eckstein, Concurs T.Lalescu, 2008
235. Determinaţi funcţiile f : ℕ∗ → ℕ∗ pentru care f (1) = 1 şi
f (n + m) = f (n) + f (m) + nm , ∀n, m ∈ ℕ∗ .
Mihai Chiş
236. Fie ( an )n≥1 un şir de numere reale cu proprietatea cǎ an+1 − an ≤ 1 , ∀n ∈ ℕ∗ , iar
a1 + a2 + ... + an
( bn )n≥1 un şir definit prin bn = , ∀n ≥ 1 . Demonstraţi cǎ :
n
1
bn+1 − bn ≤ , ∀n ≥ 1 .
2
Dan Ştefan Marinescu, Viorel Cornea, OJ 2008
237. Se considerǎ mulţimea A = {1,2,3,..., n} , n ∈ ℕ, n ≥ 6 . Arǎtaţi cǎ A este reuniunea a
trei mulţimi disjuncte douǎ câte douǎ, cu acelaşi cardinal şi aceeaşi sumǎ a elementelor,
dacǎ şi numai dacǎ n este multiplu de 3.
OJ 2008
238. Fie ABCD un patrulater inscriptibil. Se noteazǎ cu P punctul de intersecţie a
dreptelor AD şi BC, iar cu Q se noteazǎ punctul de intersecţie a dreptelor AB şi CD. Fie E
al patrulea vârf al paralelogramului ABCE şi F intersecţia dreptelor CE şi PQ.
Demonstraţi cǎ punctele D, E , F şi Q sunt conciclice.
OJ 2008
( )
239. Determinaţi funcţiile f : ℕ → ℕ pentru care f x 2 + f ( y ) = xf ( x) + y , ∀x, y ∈ ℕ .
Lucian Dragomir, ON 2008
 1 1 1 1 1 1  ∗
240. Arǎtaţi cǎ: n ⋅  1 + + + ... +  ≥ (n + 1) ⋅  + + ... +  , ∀n ∈ ℕ .
 2 3 n 2 3 n +1
Lucian Dragomir, OJ 2008, clasa a VII a
241. O succesiune de patru cifre zecimale pare în care nici o cifrǎ nu apare de trei sau
patru ori se numeşte succesiune admisibilǎ.
a) Determinaţi numǎrul de succesiuni admisibile ;
b) Pentru fiecare numǎr natural n , n ≥ 2 , notǎm cu d n numǎrul de posibilitǎţi de a
completa un tablou cu n linii şi 4 coloane cu cifre pare, respectând condiţiile urmǎtoare :
( i ) fiecare linie este o succesiune admisibilǎ ;
( ii ) succesiunea admisibilǎ 2, 0, 0, 8 ocupǎ o singurǎ linie a tabloului.

28
d n +1
Determinaţi valorile lui n pentru care este numǎr întreg.
dn
Lucian Dragomir, Nicolae Stǎniloiu, ONM 2008, clasa a VIII a
242. Pe laturile AB şi AC ale triunghiului ABC se considerǎ punctele D şi respectiv E
astfel încât DA + DB + EA + EC = 0 . Dacǎ T este intersecţia dreptelor DC şi BE,
determinaţi numǎrul real α pentru care TB + TC = α ⋅ TA .
OJ 2009, Gazeta Matematicǎ
243. Determinaţi funcţiile f : ℕ∗ → ℕ∗ pentru care
f ( x + y ) + f ( x) 2 y + f ( x)
= , pentru orice x, y ∈ ℕ∗ .
2x + f ( y) f ( x + y) + f ( y)
Lucian Dragomir, OJ 2009
244. Determinaţi numerele naturale n care satisfac simultan proprietǎţile:
n
a)   este un numǎr natural de trei cifre, toate cele trei cifre fiind egale;
9
 n + 36 
b)   este un numǎr natural de patru cifre, cifrele fiind 2, 0, 0, 9, nu neapǎrat în
 4 
aceastǎ ordine.
Lucian Dragomir, ONM 2009, clasa a VIII a
1 + an
245. Se considerǎ un şir ( an )n≥1 de numere reale, definit prin an+1 = , ∀n ∈ ℕ∗ .
1 + an 2

1
Arǎtaţi cǎ, dacǎ a1 ∈ ( 0, 2 ) , atunci an+1 − 1 ≤ n , ∀n ∈ ℕ∗ .
2
Lucian Dragomir, Shortlist ONM, 2009
246. Determinaţi numerele întregi x,y,z,t pentru care x + y + z = 3t −1 şi x 2 + y 2 + z 2 = t 3 .
Adriana şi Lucian Dragomir
247. Arǎtaţi cǎ un triunghi cu lungimile laturilor a, b, c , cu perimetrul egal cu 3 şi în care
a + b − c + b + c − a + c + a − b = 3 , este echilateral.
Adriana şi Lucian Dragomir
248. Se considerǎ o funcţie f : ℕ → ℕ care satisface urmǎtoarele proprietǎţi:
a) f ( xy ) = f ( x) + f ( y ) , ∀x, y ∈ ℕ ;
b) f (n) = 0 pentru orice n care are suma cifrelor egalǎ cu 10.
Calculaţi f (2009).
Adriana şi Lucian Dragomir
249. Determinaţi funcţiile f : ℕ → ℕ cu proprietatea cǎ
f ( x + y ) + f ( x) f ( y ) = f ( xy ) + f ( x) + f ( y ) , ∀x, y ∈ ℕ .
Lucian Dragomir

29
250. Un numǎr de şase cifre ( în scriere zecimalǎ) se numeşte olimpic dacǎ patru dintre
cifrele sale sunt 2, 0, 0, 9. Determinaţi câte numere olimpice au produsul cifrelor nenule
egal cu 72.

Adriana şi Lucian Dragomir, Concurs TM Mate Internaţional, 2009

Clasa a X a

1. Arătaţi că dacă a, b, c ∈ ℚ astfel încât a 3 3 + b 3 4 + c 3 5 = 0 ,atunci a = b = c = 0.

n +1
2. Arătaţi că dacă n ∈ ℕ, n ≥ 3 ,atunci este adevărată inegalitatea n + 1 ≤ n n.

3. Determinaţi x ∈ ℝ pentru care are loc egalitatea 3 x −1 8−3 x


( − x )x = 5x 2x .
C.Ottescu,L.Pîrşan,OJ 1973

4. Arătaţi că 3
9 + a 5 + 3 9 + a 5 = 3 dacă şi numai dacă a ∈ {−4, 4}.
Olimpiadă Timiş 2000

x2 + b + 1
5. Determinaţi a, b ∈ ℝ, a ≤ 1, b > 0 pentru care ecuaţia 3
a+x +3a−x = are
x +b2

soluţii reale.
Olimpiadă Constanţa 2001
a b c
6. Dacă a, b, c ∈ ℝ +* ,arătaţi că + 2⋅ + 3 ⋅ 3 ≥ 6.
b c a

Olimpiadă Suceava 2001


7. Ordonaţi crescător numerele a = 3 + 4, b = 2 + 5, c = 1 + 3 6.
3 3 3 3

Olimpiadă Teleorman 2001


3 3
8. Rezolvaţi ecuaţia 7 x + 1 + 8 + x − x 2 + x 2 − 8 x − 1 = 2.
3

Gheorghe Andrei,OL Constanţa 2002


 3 x = 1 + 4 y
9. Determinaţi numerele reale x şi y care satisfac : 
 x + y = 513
3 4

Gheorghe Ciorăscu,OL Neamţ 2003


10. Arătaţi că pentru orice numere reale pozitive a şi b este adevărată inegalitatea :
a 3b 1 1
3 + ≤ 3 2(a + b)  +  .
b a a b
Concurs Grigore Moisil 2003
30
11. Fie En = 2 + 2 + ... + 2 + 3 6 + 3 6 + ... + 3 6 , unde am considerat n

a) Calculaţi [ En ] ; b) Arătaţi că {En } > .


1
radicali, n ∈ ℕ*.
5
Gheorghe Andrei,OL Constanţa 2004
12. Determinaţi perechea ( x, y ) de numere reale pentru care avem:
 2a , pentru a ≥ 0
x 2 + a n + n+1 y 2 + a n+1 = 
n
.
 0 , pentru a < 0
S.Brener,ON 1976
13. Fie x ≥ 2 un număr real oarecare.Stabiliţi care număr este mai mare:
a = log x ( x + 3) sau b = log x +1 ( x + 4).
Liviu Pîrşan,ON 1972
x −2 x 2 −3 x + 2
14. Dacă A > 1, B > 1, rezolvaţi ecuaţia A +B = 2.
C.Oprişan, ON 1974
15. Fie a, b > 0, c = ab . Rezolvaţi ecuaţia : 2a = b + c . x x x

OJ 1975
16. Există numere iraţionale a şi b pentru care a ∈ ℕ ? b

Laurenţiu Panaitopol, ON 1975


1 1 1
17. Rezolvaţi ecuaţia : + = , a > 0, a ≠ 1.
2log x + a a 2log x − a a log 2 2 a
x −a

P.Toma, OJ 1976
18. Arătaţi că funcţia f : ℝ → ℝ, f ( x) = 6 − 4 ⋅ 3 − 2 + 8 nu este injectivă.
x x x

Gazeta Matematică
19. Rezolvaţi inecuaţia : xlog 4 ( x + 2) −log 2 x < 1.
OJ 1981
( ) ( )
x x
20. Rezolvaţi inecuaţia : 3 − 2 2 +1≤ 6 2 −1 .
Ion Cheşcă , OL Călăraşi 1984
 y
 x − y = ln
2 2
21. Rezolvaţi sistemul de ecuaţii:  x .
 2 x 2 − 3 xy + 2 y 2 = 9

Mircea Ganga , OL Prahova 1986
22. Arătaţi că : ln x ⋅ ln y < ln ( x + y ), ∀x, y ∈ ℝ, x > 1, y > 1.
2

Mircea Ganga , OL Prahova 1984


23. Arătaţi că pentru orice n natural nenul este adevărată inegalitatea
3
lg(n + 1) > + lg n.
10n

31
3n  1 1 1
Folosind eventual acest rezultat , arătaţi că lg(n !) >  + + ... +  .
10  2 3 n
OJ 1981
24. Determinaţi x, y ∈ ℕ care satisfac 3 − y = 1. x 3

Titu Andreescu, OJ 1982


25. Dacă a ∈ ( 0,1) este un număr fixat,rezolvaţi ecuaţia x = a . ax xa

I.V.Maftei,Titu Andreescu, OJ 1983

26. Determinaţi numerele reale strict pozitive x, x1 , x2 , x3 ,..., xn pentru care este adevărată
n n n
x
egalitatea ∑ lg( x ⋅ xk ) + ∑ lg xk
= ∑ lg xk .
k =1 k =1 k =1
Adrian Ghioca , OJ 1983
27. Determinaţi minimul expresiei
 1  1  1
E = log x1  x2 −  + log x2  x3 −  + ... + log xn  x1 −  ,
 4   4   4
1 
dacă x1 , x2 , x3 ,..., xn ∈  ,1 .
4 
Titu Andreescu, ON 1983

28. Dacă f : ℝ → ℝ este o funcţie crescătoare cu proprietatea că


f ( f ( x)) = 2 x , ∀x ∈ ℝ ,arătaţi că există a ∈ ℝ astfel încât f (a) < 0.
Marcel Chiriţă, OJ 1985
29. Rezolvaţi ecuaţia : 4 x + 9 x + 16 x = 6 x + 8 x + 12 x.
Marcel Chiriţă,OL Giurgiu 1985
log3 x + log 2 y = 2
30. Rezolvaţi sistemul de ecuaţii: 
 3 − 2 = 23
x y

I.V.Maftei,S.Rădulescu, OJ 1986
31. Rezolvaţi ecuaţia 8 + 27 + 64 + 125 = 24 + 30 + 40 x + 60 x
x x x x x x

I.V.Maftei, ON 1986
1
32. Rezolvaţi ecuaţia : 2 x −1 + 2 x = 3.
Marius Cavachi,OL Dolj 1989

33. Rezolvaţi ecuaţia : 2 x + 3x + 6 x = x 2 .


Laurenţiu Panaitopol, OJ 1990
34. Arătaţi că funcţia f : ℕ − {1} → ℝ, f ( x) = log x +1 x este strict crescătoare.
*

Mihai Dicu, OL Dolj 1985


32
 2 x + y +1 = 22 x + 2

35. Rezolvaţi sistemul : 2 y + z +1 = 22 y + 2 .
 z + x +1
 2 = 22 z + 2
V.Constantin , OL Suceava

36. Determinaţi funcţiile f : ( 0, ∞ ) → ℝ care satisfac simultan condiţiile:


a) f ( x) ≤ ln x, ∀x > 0;
b) f ( xy ) ≤ f ( x) + f ( y ), ∀x, y > 0.
OL Botoşani 1994
37. Dacă a > 0, a ≠ 1, arătaţi că nu există funcţii f : ℝ → ( 0, ∞ ) care satisfac
f ( f ( x)) = a x , ∀x ∈ ℝ.
Mihai Chiş,Concurs Traian Lalescu 1994
38. Dacă numerele a, b, c, d ∈ ℝ + − {1} , x, y, z , t ∈ ℝ satisfac
1 1 1 1
a x = bcd , b y = cda, c z = dab, d t = abc, arătaţi că ( + + + )∈ℕ .
1+ x 1+ y 1+ z 1+ t
Florin Rotaru,OJ Călăraşi 1993
39. Fie a, b, c ∈ ( 0,1) , x, y, z ∈ ℝ care satisfac a x = bc, b y = ca, c z = ab. Arătaţi că:
x + y + z ≥ 6.
OJ Sibiu 1996
2x 2a
40. Rezolvaţi ecuaţia : log a + log x = 0.
a+x a+x
Constantin Caragea,OJ Constanţa 1996
x x x
41. Rezolvaţi ecuaţia : 22 + 44 = 66 .
Gheorghe Silberberg,OJ Timiş 1991
1− x 2
42. Rezolvaţi ecuaţia : 2 x + 2 = 3.
Marian Andronache,Marcel Chiriţă, OJ Bucureşti 1991
43. Rezolvaţi ecuaţia : log3 x = log 2 (1 + x ).
OJ Botoşani 1994
44. Rezolvaţi ecuaţia : 9log5 ( x −1) − 5log9 ( x +3) = 4.
Mircea Ganga , OL Prahova 1993
1
45. Rezolvaţi ecuaţia : 9 + 9x
x
= 18.
Constantin Caragea,OJ Braşov 1993
7−2 x
46. Rezolvaţi ecuaţia : 11 = 3 x + 2.
OJ Botoşani 1993
47. Rezolvaţi ecuaţia : log x 2(1 − x) + log1− x 2 x = 0.

33
Dinu Şerbănescu, OJ Bucureşti 1996
48. Rezolvaţi ecuaţia : 10 + 11 + 12 = 13 + 14 .
x x x x x

OJ Arad 1993

x2 + 1
49. Rezolvaţi ecuaţia : log 2 (2 x 2 + 2) = .
x2 + 2
OJ Bacău 1993
2 = 1 − y x

50. Rezolvaţi sistemul :  2 y = 1 − z Dorel Miheţ, OJ Timiş
 z
 2 = 1 − x
1995
log 2 x = 1 − y

51. Rezolvaţi sistemul : log 2 y = 1 − z OJ Satu Mare
 log z = 1 − x
 2
1996
 32 x +1 + 32 y +1 + 32 z +1 = 81
52.Rezolvaţi sistemul : 
log3 (2 x + 1) + log3 (2 y + 1) + log3 (2 z + 1) = 3
Petre Năchilă, OL Prahova 1992
53. Rezolvaţi ecuaţia : xlog 2 3 = 1 + x.
Dorel Miheţ, OJ Suceava 1991.

(
54. Rezolvaţi ecuaţia : 1 + 2 x n
) + (1 + 2 ) −x n
= 8, n ∈ ℕ.
Mihail Neacşu, RMT 1987
1
1 x
55. Rezolvaţi ecuaţia : x⋅2x + ⋅ 2 = 4.
x
Laurenţiu Panaitopol , baraj
56. Rezolvaţi ecuaţia : 2 + 2[ ] + 2{ } = 3.
xx x

Şerban Olteanu , OL Giuirgiu 1998


1 1
x4 + x+
57. Rezolvaţi ecuaţia : 2 x4 +2 x = 8.
Ilie Neacşu , OJ Bihor 1998
58. Rezolvaţi ecuaţia : log x ( x + 1) = log a (a + 1), a > 0, a ≠ 1.
OJ Brăila 1998

59. Rezolvaţi în ℕ ecuaţia 2 − n = log 2 (sin ).
* n 2
8
OJ Caraş-Severin 1998

34
60. Rezolvaţi ecuaţia : 3log 2 x − 2log3 ( x +1) = 1.
Gheorghe Iurea , OJ Iaşi 1998
61. Rezolvaţi ecuaţia : 48 lg x
+ 60 lg x
+ 64 lg x
=x . 2

Aurel Doboşan , OJ Timiş 1998

62. Dacă a, b, c ∈ ( 0,1) sau a, b, c ∈ (1, ∞ ) ,arătaţi că


a logb ( ab) = blogc (bc ) = c log a ( ca ) dacă şi numai dacă a = b = c.
Dan Negulescu , OL Brăila 2000
(
63. Rezolvaţi ecuaţia : lg x3 + x = log 2 x. )
Marcel Chiriţă,Costel Chiteş, OL Bucureşti 2000
64. Se consideră numerele reale a, b, a > b > 1, A = log a (a − b), B = log b (b − a ).
Demonstraţi că dacă a 2 + b 2 = 3ab ,atunci A + B = 2 AB. Reciproca este adevărată ?
Dorel Miheţ, OL Caraş-Severin 2000
65. Rezolvaţi ecuaţia : 3x + 4 x + 5 x = 6 x + x − 3.
Petre Năchilă , OL Prahova 2000
66. (
Să se rezolve ecuaţia : log 4 x 4 − 4 x + 5 = ) x +1
2
x
.

Lucian Dragomir, RMT 2004

67.
(
 9 x + 9 y + 9 z 3− x + 3− y + 3− z = 3

Rezolvaţi sistemul : 
)( )
3x + y +1 + 3 y + z +1 + 3z + x +1 = 3x + 3 y + 3z
Valentin Matrosenco, OJ Argeş 2000
2 x +1
68. Rezolvaţi ecuaţia : 32 x +1 ⋅ 8 2 x + 2 = 36.
Marcel Chiriţă, OJ Argeş 2000
69. Rezolvaţi în mulţimea numerelor întregi ecuaţia 9 − 3 = 36 x. x x

Dan Negulescu , OJ Brăila 2000


log x = log3 (1 + y )
70. Rezolvaţi sistemul de ecuaţii :  2
log3 (1 + x) = log 2 y
Gheorghe Pantelimon , OJ Călăraşi 2000
71. Fie n ∈ ℕ .Rezolvaţi în mulţimea numerelor întregi ecuaţia x n + 25 = x 2 + 5n.
*

Marcel Chiriţă , OJ Neamţ 2000


2
−3 x x 2 − 3x
72. Rezolvaţi ecuaţia : 2 x + = 1, x ∈ ℝ.
2x
Sorin Rădulescu , OL Bucureşti 2001
73. Arătaţi că numerele 1.log 2 3,log 3 2 nu pot fi termeni ai unei aceleeaşi progresii
aritmetice.
Traian Duţă,OL Braşov 2001
35
74. Rezolvaţi ecuaţia : (2 + x)log 2 3 − (3 + x)log3 2 = 1.
Dan Negulescu,OL Brăila 2001
log x + log 2 (1 − y ) = 2
2 2
75. Rezolvaţi sistemul :  2
log 2 y + log 2 (1 − x) = 2
2 2

Nelu Chichirim, OL Constanţa 2001


76. Rezolvaţi ecuaţia
x x x
 1 1 1
: log 2 ( x + 1) + log3 ( x + 1) + ... + log n ( x + 1) + (n − 1) =   +   + ... +  
 2 3 n
OL Hunedoara 2001
77. Dacă a ∈ ℚ, a > 1 ,arătaţi că log a (a + 1) ∉ ℚ.
Manuela Prajea, OL Mehedinţi 2001
1
78. Rezolvaţi ecuaţia : 2 lg x
+ 8 = ( x − 8) lg 2
.
Daniel Jinga, OJ 2001
79. Rezolvaţi ecuaţia : 3 + 5 = 2 + 2 + 4 .
x x x x

Marian Ionescu, OL Argeş 2002


sin 2 x cos 2 x
80. Rezolvaţi ecuaţia : 2 −2 = cos 2 x.
Traian Tămâian, OL Brăila 2002
2 − 2 = y − x x y
81. Rezolvaţi în numere întregi sistemul : 
 2 + 2 = 2 xy
x y

Lucian Dragomir, OL Caraş-Severin 2002


 4 − 18 ⋅ 3 + 98 = 0
x y

82. Rezolvaţi sistemul :  9 y − 8 ⋅ 2 z − 49 = 0
 z
4 − 16 ⋅ 2 + 112 ≤ 0
x

Paul Băiatu, OL Giurgiu 2002


83. Dacă a, b, c ∈ (1, ∞ ) ,arătaţi că log a 4 b + logb 4 c + log c 4 a ≥ log a b + logb c + log c a.
Steluţa şi Mihai Monea, OL Hunedoara 2002
84. Rezolvaţi ecuaţia : log14 ( x + x + x ) = log 64 x.
3 6

Aurel Doboşan , OL Sibiu 2002


85. Fie a şi b două numere reale care satisfac 3a + 13b = 17 a şi 5a + 17b = 11b.
Arătaţi că a < b.
Cristinel Mortici , OJ 2002
n2
86. Demonstraţi inegalitatea : log 2 2 3 + log 32 4 + ...log n 2 (n + 1) ≥ .
n +1
Augustin Drăgan, OL Bihor 2003

36
87. Rezolvaţi ecuaţia : xlog 2 3 − 3log 2 x = 2 − x.
Aurel Bârsan, OL Braşov 2003
a + a +1
2
Determinaţi a ∈ ℝ pentru care ecuaţia 2 x −1 + 2 x −1
2
88. = are soluţii întregi.
a2 + 1
Petru Răducanu, OL Iaşi 2003
1
89. Rezolvaţi ecuaţia : 4 x + 1 − 4 x 2 = .
2
OL Mehedinţi 2003
n +1 1
90. Arătaţi că log n ! > , ∀n ∈ ℕ, n ≥ 2.
2 n
OL Mehedinţi 2003

91. Arătaţi că există o infinitate de triplete ( x, y, z ), x, y, z ∈ (1, ∞ ) pentru care


y+z z+x x+ y
log x + log y + log x = 3.
2 2 2
OL Olt 2003
πx 2
− 4 x +3
92. Rezolvaţi ecuaţia : tg = 2x .
x +3
2

Dan Popescu , OL Suceava 2003


93. Se consideră numerele reale a > c > d > b > 1, pentru care ab > cd . Arătaţi că funcţia
f : [ 0, ∞ ) → ℝ, f ( x) = a x + b x − c x − d x este strict crescătoare.
Cristinel Mortici , OJ 2003
1  1 
94. Rezolvaţi ecuaţia : 4 + 4x
x 
= 2⋅ 2 + 2x
x .
 
 
Nelu Chichirim , OL Constanţa 2004
log ab c logbc a log ca 2 b
2 2
9
95. Dacă a, b, c ∈ (1, ∞ ) ,arătaţi că : + + ≥ .
a+b b+c c+a 8(a + b + c)
Marius Ghergu , OL Dolj 2004
96. a) Demonstraţi că : log3 7 ⋅ log3 11 > log52 10.
b) Rezolvaţi ecuaţia : 9 x + ( x − 3) ⋅ 3x + 2 = 4 x + ( x − 1) ⋅ 2 x + x.
Nicolae Papacu , OL Bucureşti 2005
3 3 3 3
97. Rezolvaţi ecuaţia: 3 ⋅ 2 x − 6 + 2 ⋅ 2 x − 4 = 3 ⋅ 2 x − 4 + 2 ⋅ 2 x − 6.
Marcel Chiriţă , OL Bucureşti 2005
98. Rezolvaţi ecuaţia : xlog 2 a − a ⋅ log 2 x + x − 2 = 0.
Dan Popescu , OL Suceava 2005

37
 x + y + z = 3
99. Rezolvaţi sistemul :  2
y2 z2
 2 + 2 + 2 = 6
x

Laurenţiu Panaitopol,concurs 2005



log3 x + log 5 y = 2

100. Rezolvaţi sistemul :  3 y − 5 x ≥ 118
 log x − log ≤ 0
 1 1
 3 5
Vasile Berinde,Concurs Grigore Moisil 2005

101. Arătaţi că pentru orice n ∈ ℕ există x ∈ (1, ∞ ) astfel încât [ log x 3] − [ log x 2] = n.
Gheorghe Iurea, ON 1993
x ⋅ y = z2

102. Rezolvaţi în mulţimea numerelor complexe sistemul :  y ⋅ z = x 2

 z ⋅ x = y
2

Marius Burtea , OL Teleorman


103. Fie a, b, c ∈ ℂ* ,distincte două câte două.Arătaţi că dacă
a 2 = bc, b 2 = ca, c 2 = ab, atunci a,b,c sunt afixele vârfurilor unui triunghi echilateral.
Constantin Cocea , OL Iaşi 1990
104. Fie a, b, c ∈ ℂ astfel încât Im(ab) = Im(bc) = Im(ca ) ≠ 0. Arătaţi că:
2 2 2
(
a − b + b − c + c − a = 3⋅ a + b + c
2 2 2
).
Marian Andronache,OL Bucureşti 1988
2π 2π
105. Fie ε = cos + i ⋅ sin . Arătaţi că dacă z ∈ ℂ satisface z − ε ≤ 1 şi
3 3
z − ε 2 ≤ 1, atunci z ≤ 1.
Constantin Cocea , OL Iaşi 1987
106. Fie a, b, c ∈ ℂ .Arătaţi că dacă Re(az + bz + c) ≥ 0, ∀z ∈ ℂ, atunci a = b = 0.
2

Jenică Crînganu, OL Galaţi


107. Dacă A1 A2 ... An este un poligon regulat înscris în cercul
n
n 1
C (0,1), M ∈ (OA1 , A1 ∈ (OM ) ,arătaţi că : ∑ MA OM
. >
k =1 k
Mihai Piticari , Test tabără naţională 1983
108. Fie a, b, c ∈ ℂ ,nu toate reale,astfel încât a = b = c = 1 şi (2(a + b + c) − 3abc) ∈ ℝ .

38
π
Arătaţi că : max {arg a,arg b,arg c} ≥ .
6
Titu Andreescu,Test tabără naţională 1986
1 1 1
109. Fie a, b, c ∈ ℂ* cu a = b = c . Arătaţi că: 0 ≤ ( a + b + c )  + +  ≤ 9.
a b c
Gheorghe Andrei , OL Constanţa 1985
110. Fie z1 , z2 , z3 ∈ ℂ ,disincte,astfel încât z3 − z2 + ε ( z1 − z3 ) + ε 2 ( z2 − z1 ) = 0, unde
2π 2π
ε = cos + i ⋅ sin . Arătaţi că numerele complexe
3 3
z1 + z2 − z3 , z2 + z3 − z1 , z1 + z3 − z2 sunt afixele vârfurilor unui triunghi echilateral.
OL Constanţa 1987
111. Se consideră numerele complexe nereale a,b,p,q astfel încât
( )
n
a = b , p = q , a 2 ≠ b 2 . Arătaţi că ecuaţia p ( az + b ) + q az + b
n
= 0, n ∈ ℕ* are
toate rădăcinile reale.
Silviu Boga, OJ Suceava 1993
112. Fie a, b, c ∈ ℂ afixele vârfurilor unui triunghi ABC.Notând u = a − b, v = c − b ,arătaţi
că triunghiul ABC este dreptunghic în A dacă şi numai dacă Re(u ⋅ v) = 0.
Dan Ştefan Marinescu , OL Hunedoara 1994
113. Arătaţi că dacă z este un număr complex cu modulul 1 , atunci
13
3 ≤ 1 + z + 1 − z + z2 ≤ .
4
Ovidiu Pop , OJ Satu Mare 1996
114. Fie x, y , z ∈ ℂ astfel încât x = 3, y = 4, z = 5, x + y + z = 0.
a) Arătaţi că 16 x 2 + 9 y 2 = 0;
b) Determinaţi lungimile laturilor triunghiului ale cărui vârfuri au afixele x , y , z .
Laurenţiu Panaitopol , OL Bucureşti 1995
b
115. Fie a, b ∈ ℂ cu a + b = a = b . Calculaţi .
*
a
Bogdan Enescu , OJ Brăila 1995
116. Se consideră ecuaţia az 2 + bz + c = 0, a, b ∈ ℤ, a ≠ 0. Arătaţi că dacă f (i ) < 1 ,atunci
ecuaţia nu are rădăcini reale.
Gheorghe Andrei , OL Constanţa 1991
117. Demonstraţi că numerele complexe distincte z1 , z2 , z3 sunt afixele vârfurilor unui
( ) (
triunghi dreptunghic dacă şi numai dacă ( z1 − z2 ) z1 − z3 + z1 − z2 ) ( z1 − z3 ) = 0.
Gheorghe Andrei , OL Constanţa 1994
118. Fie z1 , z2 , z3 ∈ ℂ .Dacă z1 z2 z3 , z2 z1 z3 , z3 z1 z2 ∈ ℝ +* ,arătaţi că punctele de afixe
z1 , z2 , z3 sunt colineare.
39
Constantin Caragea , OL Constanţa 1995
119. Arătaţi că dacă n ∈ ℕ ,atunci
*

π 1
sin ≥ . MA1 + MA2 + ... + MAn ≥ OA1 + OA2 + ... + OAn .
2n n
Marius Cavachi , OJ Constanţa 1993

120. Fie A1 A2 ... An un poligon cu proprietatea că există un punct O în planul său astfel

încât m(∡A1OA2 ) = m(∡A2OA3 ) = ... = m(∡An−1OAn ) = m(∡AnOA1 ) = . Arătaţi că
n
pentru orice punct M din planul poligonului există inegalitatea
Cristinel Mortici , OL Constanţa 1998
121. Fie z1 , z2 , z3 ∈ ℂ − ℝ ,distincte două câte două şi având acelaşi modul.Să se arate că
dacă z1 + z2 z3 , z2 + z1 z3 , z3 + z1 z2 sunt reale ,atunci z1 z2 z3 = 1.
Laurenţiu Panaitopol , OJ 1979
122. Fie z1 , z2 , z3 ∈ ℂ* , z1 = z2 = z3 .
a) Arătaţi că există numerele complexe a , b astfel încât
z2 = az1 , z3 = bz1 , a = b = 1.
b) Rezolvaţi în raport cu una dintre necunoscutele a şi b ecuaţia
a 2 + b 2 − ab − a − b + 1 = 0.
c) Folosind eventual rezultatele anterioare,arătaţi că dacă
z12 + z2 2 + z32 = z1 z2 + z2 z3 + z3 z1 ,atunci z1 = z2 = z3 sau imaginile
geometrice ale numerelor z1 , z2 , z3 sunt vârfurile unui triunghi echilateral.
Ioan Tomescu , OJ 1982
123. Fie z1 , z2 , z3 ∈ ℂ, z1 = z2 = z3 = r , z3 ≠ z2 . Arătaţi că :
z1 − z2 ⋅ z1 − z3
min az2 + (1 − a) z3 − z1 = .
a∈ℝ 2r
Dorin Andrica , ON 1983
124. Fie ecuaţia ax + bx + c = 0, a, b, c ∈ ℂ astfel încât :
2

arg a + arg c = 2arg b, a + c = b . Arătaţi că cel puţin o rădăcină a ecuaţiei are


modulul 1.
Laurenţiu Panaitopol , OJ 1973
125. Fie a, b, c ∈ ℂ afixele vârfurilor unui triunghi ABC.Arătaţi că ABC este echilateral
dacă şi numai dacă ecuaţia az 2 + bz + c = 0 are fie rădăcina
2π 2π 4π 4π
z1 = cos + i sin fie z2 = cos + i sin .
3 3 3 3
OJ 1985
126. Fie z1 , z2 , z3 ∈ ℂ cu z1 + z2 + z3 ≠ 0, z12 + z2 + z3 = 0 şi z1 = z2 = z3 = 1.
2 2

40
Arătaţi că : z1 + z2 + z3 = 2.
Florin Vulpescu-Jalea , ON 1985
127. Arătaţi că nu există trei numere z1 , z2 , z3 ∈ ℂ cu z1 = z2 = z3 = 1 care să verifice
relaţia : z13 + z23 + z33 = 3 ⋅ (2 − 2i − z1 + z2 + z3 ).
I.V.Maftei,S.Rădulescu , OJ 1986
128. Fie A o submulţime a mulţimii numerelor complexe cu proprietăţile :
a) A conţine orice număr z ∈ ℂ, cu z = 1;
b) Pentru orice z1 , z2 ∈ A avem z1 + z2 ∈ A.
Arătaţi că A = ℂ.
Marcel Ţena , ON 1986
n  n z 
129. Fie z1 , z2 ,..., zn ∈ ℂ* având acelaşi modul. Arătaţi că numărul ∑  ∑  are partea
j
 z 
j =1  k =1 k 
n
reală nulă dacă şi numai dacă ∑ zk = 0.
k =1
OJ 1987
2π 2π
130. Fie p > 2 un număr prim şi z ∈ ℂ, z = cos + i sin . Determinaţi numerele
p p
1
raţionale a0 , a1 ,..., a p −2 pentru care = a0 + a1 z + ... + a p −2 z p − 2 .
1− z
Mircea Becheanu,Laurenţiu Panaitopol, ON 1987
131. Fie n > 2 un număr natural şi z ∈ ℂ − {1} cu z n = 1.
2
a) Arătaţi că 1 − z > ;
n −1
b) Arătaţi că pentru orice k ∈ ℤ , k nedivizibil cu n , are loc inegalitatea
kπ 1
sin > .
n n −1
Mircea Becheanu , OJ 1988
132. Fie x, y ∈ ℂ, n ∈ ℕ, n ≥ 2 astfel încât x n− 2 ⋅ y = 2 şi x n = y n = x + y.
Arătaţi că x = y.
Maria Elena Panaitopol , OL Bucureşti 1998

133. Fie ABC un triunghi într-un sistem de axe de coordonate cu originea în centrul
cercului circumscris triunghiului . Dacă a , b ,c sunt afixele vârfurilor triunghiului dat
, arătaţi că :
a + b + b + c + c + a ≤ 3R ,unde R este raza cercului circumscris triunghiului
dat.
OL Vaslui 1998
41
134. Fie z1 , z2 , z3 ∈ ℂ cu z1 = z2 = z3 = 1 şi z1 + z2 + z3 = 1. Arătaţi că :
n n n
z13 + z23 + z33 = 1, ∀n ∈ ℕ.
OJ Harghita , 1998
135. Determinaţi numerele complexe z ştiind că există un număr natural n ≥ 2 pentru
1 1
care are loc egalitatea z + = z n + n = 2.
z z
Octavian Purcaru , OJ Orahova 1998
136. Dacă a, b, c ∈ ℂ cu a = b = c = R > 0 ,arătaţi că
a + b − c + b + c − a + c + a − b ≥ 12r ,unde r este raza cercului înscris în triunghiul
cu vârfurile de afixe a , b , c .
OJ Satu-Mare , 1998
137. Dacă z1 , z2 , z3 , z4 ∈ ℂ sunt distincte şi satisfac egalităţile :
z1 + z3 = z2 + z4 , z1 + iz3 = z2 + iz4 , arătaţi că există z ∈ ℂ astfel încât
z − z1 = z − z2 = z − z3 = z − z4 .
OJ Timiş , 1998

138. Arătaţi că dacă z ∈ ℂ, z 2 + 1 = 2 ⋅ z + 1 ,atunci z ≤ 7.


Virgil Nicula , OL Cluj 2000
139. Determinaţi numerele z1 , z2 ,..., z8 ∈ ℂ ştiind că
z2 = 1 + iz1 , z3 = 1 + iz2 ,..., z8 = 1 + iz7 , z1 = 1 + iz8 .
Dan Ştefan Marinescu,Ioan Şerdean, OL Hunedoara 2000

140. Fie z1 , z2 , z3 ∈ ℂ* cu z1 = z2 = z3 şi z1 + z2 = z 2 + z3 = z3 + z1 .Arătaţi că :


z1 − z2 = z 2 − z3 = z3 − z1 .
Viorel Cornea , OL Hunedoara 2000
1 1
141. Fie z ∈ ℂ. Arătaţi că : z 2 + 2
≤2⇒ z + ≤ 2.
z z
Virgil Nicula , Marcel Chiriţă, OJ Argeş 2000
142. Fie a, b ∈ ℂ* şi z1 , z2 rădăcinile ecuaţiei x 2 + ax + b = 0. Arătaţi că
a2 a2
z1 = z2 ⇔ ∈ ℝ şi 0 < ≤ 4.
b b
OJ Bucureşti 2000
143. Demonstraţi că pentru orice n ∈ ℕ, n ≥ 2 şi orice z ∈ ℂ are loc inegalitatea :
n z ≤ n Re z + n Im z .
Mircea Berca , OJ Giurgiu 2000
144. Fie z1 , z2 , z3 ∈ ℂ cu z1 + z2 + z3 ≠ 0, z12 + z2 + z3 = 0 şi z1 = z2 = z3 = 1.
2 2

42
Arătaţi că : z13 + z23 + z33 = 1.
Gheorghe Szöllösy , OJ Hunedoara 2000
145. Fie M o mulţime de numere complexe cu proprietatea că pentru orice
x
x, y ∈ M ⇒ ∈ M . Arătaţi că dacă mulţimea M are n elemente , atunci M este mulţimea
y
rădăcinilor de ordinul n ale unităţii.
OJ Maramureş , 2000
146. Fie z1 , z2 , z3 ∈ ℂ afixele vârfurilor triunghiului ABC şi a ∈ ℂ, a ≠ z1 + z2 + z3 . Arătaţi
că dacă punctele M , N , P au afixele
z12 − az1 − z2 z3 , z22 − az2 − z1 z3 , respectiv z32 − az3 − z2 z1 , atunci triunghiurile ABC şi
MNP sunt asemenea.
Valentin Matrosenco,Marian Andronache , OL Bucureşti 2001
147. Se consideră patrulaterul convex ABCD cu ( BC ) ≡ (CD ) şi punctul M situat de
aceeaşi parte cu D faţă de dreapta AB astfel încât ( AM ) ≡ ( BM ), m(∡AMB ) = m(∡BCD ).
Ştiind că AD = 3 ⋅ MC ,calculaţi m(∡BCD).
Marian Andronache , OL Bucureşti 2001
148. { }
Fie A = z ∈ ℂ / z = x + i ⋅ 2 , x ∈ ℝ .
x

a) Arătaţi că ∃z1 , z2 , z3 ∈ A astfel încât afixele lor să fie vârfurile unui triunghi
isoscel ;
b) Arătaţi că ∀z1 , z2 , z3 ∈ A afixele lor nu pot fi vârfurile unui triunghi
echilateral.
Dan Ştefan Marinescu,Ioan Şerdean, OL Hunedoara 2001

149. a) Fie z1 , z2 ∈ ℂ cu z1 + z2 > 0 şi z1 = z2 . Demonstraţi că z1 ⋅ z2 > 0.


b) Rezolvaţi inecuaţia z 2 + z ≤ 0, z ∈ ℂ.
OL Timiş 2001
150. Se spune că perechea de numere complexe ( z1 , z2 ) ∈ ℂ × ℂ are proprietatea (P) dacă
* *

există un număr real a ∈ [ −2, 2] astfel încât z12 − az1 z2 + z22 = 0. Arătaţi că dacă ( z1 , z2 )

( )
are proprietatea (P),atunci pentru orice număr natural n perechea z1n , z2n are această
proprietate.
Dorin Andrica , OJ 2001
151. Se consideră pentagonul inscriptibil ABCDE.Notăm cu
H1 , H 2 , H 3 , H 4 , H 5 ortocentrele triunghiurilor ABC,BCD,CDE,DEA,EAB şi cu
M1 , M 2 , M 3 , M 4 , M 5 mijloacele laturilor DE,EA,AB,BC şi respectiv CD.Arătaţi că
dreptele H1M1 , H 2 M 2 , H 3 M 3 , H 4 M 4 , H 5 M 5
sunt concurente.

43
Dinu Şerbănescu , OJ 2001
152. Fie a, b, c ∈ ℂ şi D = { z ∈ ℂ / z ≤ 1}. Arătaţi că dacă pentru orice z ∈ D avem
*

( az 2
)
+ bz + c ∈ D ,atunci {a, b, c} ⊂ D.
Manuela Prajea , OL Botoşani 2002
153. Rezolvaţi ecuaţiile :
a) z − a + z − b = b − a, z ∈ ℂ ,unde a , b sunt numere reale fixate cu a < b;
b) z + z − 1 + z − 2 + z − 3 = 4, z ∈ ℂ.
Sorin Rădulescu,Petruş Alexandrescu , OL Bucureşti 2002

154. Numerele complexe distincte a , b , c sunt afixele vârfurilor unui triunghi dreptunghic
isoscel ABC,cu unghiul drept A.Demonstraţi că : ( a − b ) + ( a − c ) = 0. Reciproca este
2 2

adevărată ?
OL Buzău , 2002
155. Să se găsească ecuaţia dreptei pe care sunt situate imaginile geometrice ale
rădăcinilor ecuaţiei z 2 − 2iz − 1 − 2ia 2 = 0, a ∈ ℝ.
Dan Brânzei , OL Caraş-Severin 2002
156. Fie z ∈ ℂ, z = 1.
a) Arătaţi că ∀n ∈ ℕ* : 1 − z 2 + 1 + z 3 + ... + 1 − z 2n + 1 + z 2n +1 ≥ n ⋅ 1 + z ;
b) Arătaţi că ∀α ∈ ℝ : sin 2α + cos3α + ... + sin 2nα + cos(2n + 1)α ≥ n ⋅ cos α .
Dan Ştefan Marinescu , OL Hunedoara 2002
 x( x − y )( x − z ) = 3

157. Rezolvaţi în mulţimea numerelor complexe sistemul :  y ( y − x)( y − z ) = 3
 z ( z − x)( z − y ) = 3

Mihai Piticari , OJ 2002
158. Determinaţi locul geometric al punctelor M de afix z pentru care punctele A de afix 1
, M şi N de afix z 3 sunt coliniare.
Adriana Caţaran , OL Braşov , 2003
159. Fie ABCD un patrulater convex şi M mijlocul laturii (CD).Dacă AB = BC + AD şi
BM ⊥ AM ,arătaţi că BC // AD.
Laurenţiu Panaitopol , OL Bucureşti 2003
160. Fie a , b , c , d numere complexe de acelaşi modul astfel încât a + b + c = d . Arătaţi că
unul dintre numerele a , b , c este egal cu d.
Marcel Ţena , OL Bucureşti 2003
161. Fie triunghiul ABC şi punctele M ∈ ( BC ) , N ∈ ( CA ) , P ∈ ( AB ) astfel încât
BM CN AP
= = = k > 0, k ≠ 1. Arătaţi că dacă triunghiul MNP este echilateral,atunci şi
MC NA PB
triunghiul ABC este echilateral.
44
Cezar Corneliu-Stoica , OL Constanţa 2003
162. Fie triunghiul ABC şi punctele M ∈ ( BC ) , N ∈ ( CA ) , P ∈ ( AB ) astfel încât
BM CN AP
= = . Arătaţi că dacă ortocentrele triunghiurilor ABC şi MNP coincid,atunci
MC NA PB
triunghiul ABC este echilateral.
OL Hunedoara , 2003
163. Fie a, b, c ∈ ℂ* , a ≠ 0 şi z1 , z2 rădăcinile ecuaţiei ax 2 + bx + c = 0. Dacă z1 z2 = 1 ,arătaţi
că : b + c ≥ 3 a .
Petru Răducanu , OL Iaşi 2003
164. Fie x , y , z afixele vârfurilor A , B , respectiv C ale unui triunghi ABC înscris în
cercul C (O,1). Arătaţi că dacă yz (b + c) + zx(c + a) + xy (a + b) = 0 ,atunci triunghiul ABC
este echilateral.
Marian Ursărescu , OL Neamţ 2003
165. a) Dacă ABC este un triunghi şi M un punct în planul său,arătaţi că
AM ⋅ sin A ≤ BM ⋅ sin B + CM ⋅ sin C ;
b) Fie A1 , B1 , C1 puncte pe laturile (BC),(AC),respectiv (AB) ale triunghiului ABC
astfel
încât unghiurile triunghiului A1B1C1 sunt,în această ordine,de măsuri α , β , χ . Arătaţi că
∑ AA1 ⋅ sin α ≤ ∑ BC ⋅ sin α .
Dan Ştefan Marinescu , OJ 2003
166. Se consideră hexagonul inscriptibil
ABCDEF . Fie H1 , H 2 , H 3 , H 4 , H 5 , H 6 ortocentrele triunghiurilor
ABC , BCD, CDE , DEF , EFA, respectiv FAB. Arătaţi că dreptele
H1H 4 , H 2 H 5 , H 3 H 6 sunt concurente.
Mihai Monea , OL Hunedoara 2004
 π
Fie x, y ∈  0,  . Arătaţi că dacă egalitatea ( cos x + i sin y ) = cos nx + i sin ny este
n
167.
 2
adevărată pentru două numere naturale consecutive,atunci este adevărată pentru toate
numerele naturale n .
Dinu Şerbănescu , OJ 2004
168. Fie ABC un triunghi cu m(∡A) < 900. În exteriorul triunghiului ABC se consideră
punctele D şi E astfel încât DA = DB, EA = EC , m ( ∡ADB ) = m ( ∡AEC ) = 2m ( ∡A ) .
Demonstraţi că simetricul punctului A faţă de mijlocul segmentului (DE) este centrul
cercului circumscris triunghiului ABC.
Concurs Myller , 2005
169. Se consideră punctele necoliniare A , B ,C având afixele a ,b , respectiv a – b , astfel
încât triunghiul ABC este isoscel.Determinaţi locul geometric al punctului A dacă B este
fix.
Dan Brânzei , Concurs Unirea 2005

45
170. Fie f : ℝ → ℝ o funcţie cu proprietatea : f ( f ( x)) = x 2 − x + 1, ∀x ∈ ℝ. Arătaţi că:
a) f (1) = 1;
b) funcţiile f , g : ℝ → ℝ, g ( x) = x 2 − xf ( x) + 1 nu sunt injective.
Dan Seclăman , OL Dolj , 1983
171. Determinaţi funcţiile injective f : ℝ → ℝ care satisfac
f ( f ( x) + y ) = f ( x + y ) + f (0), ∀x, y ∈ ℝ.
***
172. Arătaţi că nu există funcţii bijective f : ( 0, ∞ ) → ( 0, ∞ ) cu proprietatea că :
f ( x) + f ( x + y ) = y, ∀x, y ∈ ( 0, ∞ ) .
RMT 1981
173. Fie f : ℝ → ℝ o funcţie cu proprietatea : f ( x) f ( y ) = f ( x + y ), ∀x, y ∈ ℝ.
Arătaţi că : a) f nu este surjectivă ;
b) f este injectivă dacă şi numai dacă f (0) = 1, f ( x) ≠ 1, ∀x ∈ ℝ*.
***
174. Fie f : ℝ → ℝ astfel încât f este injectivă şi
f ( x) f (1 − x) = f (ax + b), ∀x ∈ ℝ, unde a, b ∈ ℝ.
Arătaţi că : a) a = 0;
b) f (1 − b) = 1;
c) f nu este surjectivă.
Maria Elena Panaitopol , OL Bucureşti 1983
175. Fie f : ℝ → ℝ astfel încât ( f f )( x) = − x, ∀x ∈ ℝ. Arătaţi că :
a) f este bijectivă ;
b) f nu este strict monotonă ;
c) f (0) = 0.
OL Prahova 1986
1
176. Există funcţii injective f : ℝ → ℝ astfel încât f ( x 2 ) − f 2 ( x) ≥ , ∀x ∈ ℝ ?
4
Titu Andreescu , ON 1981
177. Fie f : A → B o funcţie.Demonstraţi că următoarele afirmaţii sunt echivalente :
a) f este injectivă ;
b) oricare ar fi E , F ⊂ A, E ∩ F = ∅ ⇒ f ( E ) ∩ f ( F ) = ∅.
Emil Moldoveanu, OL Bucureşti 1985
 x , x∈ℚ
178. Fie f : ℝ → ℝ , f ( x) =  .Arătaţi că f este inversabilă şi precizaţi inversa
3 − x , x ∉ ℚ
sa.
OL Dâmboviţa 1987

46
179. a) Fie g : ℝ → ℝ o funcţie surjectivă şi f : ℝ → ℝ o funcţie astfel încât
f g = g . Arătaţi că f = 1R.
1 , x ∈ ℚ
b) Dacă g : ℝ → ℝ , g ( x) =  ,arătaţi că există f : ℝ → ℝ , f ≠ 1R astfel
0 , x ∉ ℚ
încât f g = g .
Dorel Miheţ , OL Timiş 1986
180. a) Determinaţi funcţiile injective f : ℝ → ℝ cu proprietatea că f f = f .
b) Determinaţi funcţiile surjective g : ℝ → ℝ cu proprietatea că g g = g .
Florin Vulpescu-Jalea , OL Bucureşti , 1987
181. Fie a, b, c ∈ ℝ, a ≠ 0.
a) Arătaţi că funcţia f : ℝ − ℚ → ℝ, f ( x) = ax 2 + bx + c nu este injectivă pe
ℝ−ℚ ;
b) Arătaţi că funcţia g : ℚ → ℚ, g ( x) = ax 2 + bx + c este injectivă dacă şi numai
b
dacă ∈ ℝ − ℚ.
a
Florin Vulpescu-Jalea , OL Bucureşti , 1989
182. Arătaţi că produsul a două funcţii bijective f , g : ℤ → ℤ nu poate fi o funcţie
bijectivă.
Jenică Crînganu,OL Galaţi 1990
183. Arătaţi că nu există funcţii injective care satisfac condiţia
f 2 ( x) + f ( x) + 1 ≤ 3 f ( x 2 − x), ∀x ∈ ℝ.
OL Olt , 1998
184. Fie a, b numere reale strict pozitive şi diferite de 1. Arătaţi că există o funcţie
injectivă f : ℝ → ℝ cu proprietatea că f (a x ) + f (b − x ) = 0, ∀x ∈ ℝ dacă şi numai dacă
a = b.
Romeo Ilie , OL Braşov 1998
185. a) Fie a > 0, a ≠ 1. Arătaţi că funcţia u : ℝ → ℝ, u ( x) = a (1−a ) x +1 este strict
descrescătoare pe ℝ;
b) Dacă a, b > 0 ,rezolvaţi inecuaţia a x ⋅ a1−ax + b x ⋅ b1−bx > a a + bb .
Valentin Matrosenco , OL Bucureşti 1998
186. Fie f : ℝ → ℝ 0 funcţie neinjectivă şi g : ℝ → ℝ astfel încât
f ( x + y ) = g ( f ( x) + y ), ∀x, y ∈ ℝ. Arătaţi că f este periodică.
OJ Brăila , 1998
187. Fie f : ℝ + → ℝ, f ( x) = a 2 x − 2a x + 3, a > 1. Arătaţi că f este injectivă,determinaţi
A = f ( ℝ ) şi f −1 : A → ℝ + .
OL Arad 2002

47
188. Fie a un număr real nenul fixat.Demonstraţi că nu există funcţii injective
f : ℝ → ℝ cu proprietatea că f (a sin x) + f (a cos x) = a 2 , ∀x ∈ ℝ.
Lucian Dragomir , OL Caraş-Severin 2002
189. Fie D = ℤ × ℤ = {( x, y ) / x, y ∈ ℤ}.
a) Arătaţi că funcţia f : D → D, f ( x, y ) = (3 x + 2 y, 4 x + 3 y ), ∀( x, y ) ∈ D este
bijectivă
b) Pentru a,b,c,d ∈ ℤ se defineşte funcţia
g : D → D, g ( x, y ) = (ax + by, cx + dy ), ∀( x, y ) ∈ D. Stabiliţi condiţii necesare şi
suficiente pentru ca funcţia g să fie :
(i) injectivă ; (ii) bijectivă.
Alexandru Dincă , OL Dolj , 2002
190. Fie a, b > 0, a, b ≠ 1. Se consideră o funcţie injectivă f : ℝ → ℝ astfel încât funcţia
definită prin g : ℝ → ℝ, g ( x) = f (log a x) + f (log b x) este constantă.Demonstraţi că
ab = 1 şi arătaţi că există funcţii f : ℝ → ℝ care satisfac cerinţele enunţului.
Dan Popescu , Mihai Piticari , OL Suceava 2002
x
191. a) Folosind definiţia, arătaţi că funcţia f : ( 0, ∞ ) → ℝ, f ( x) = este concavă;
x +1
b) Demonstraţi că dacă n ∈ ℕ, n ≥ 2, x1 , x2 ,..., xn ∈ ( 0, ∞ ) astfel încât
n n
1 1
∑1+ x ≥ n − 1 ,atunci ∑x ≥ n(n − 1).
k =1 k k =1 k
Lucian Tuţescu , Concurs Gh.Dumitrescu 2005
192. Arătaţi că orice funcţie f : ℚ → ℚ se poate scrie ca suma a două funcţii injective.
Ion Savu , Concurs Arhimede 2005
193. Se consideră funcţia f : [ a, b ] → ℝ, a < b, f ( x) = x − a + b − x . Arătaţi că f este
 a +b a + b 
crescătoare pe  a,  şi descrescătoare pe  , b .
 2   2 
Laurenţiu Panaitopol , ON 1975
194. {
Fiind dat un număr natural m ≥ 2 ,arătaţi că funcţia f : ℕ → ℕ, f (n) = m n ⋅ 2 este }
injectivă, { x} reprezentând partea fracţionară a numărului real x.
Constantin Niţă , OJ 1980
195. Dacă a şi b sunt numere reale , arătaţi că funcţia f : ℝ → ℝ, f ( x) = x + a + x + b nu
este injectivă.
***

196. Fie f: A → A, unde A = {a 1, a 2,..., a n} ⊂ ℕ *, cu a 1 < a 2 <... < a n, n ≥ 2, o funcţie


1
cu proprietatea că f(x) – f (y) ≥ x − y − , ∀ x, y ∈ A.
x+ y

48
Arătaţi că există k ∈ {1, 2,..., n} astfel încât f (a k) = a 2.
Cristinel Mortici, OL Constanţa. 1997
197. Fie f: [0, 1] → ℝ o funcţie cu proprietăţile:
(i) f (1) = 1;
(ii) f (x) ≥ 0, ∀ x ∈ [0, 1];
(iii) Dacă x, y şi x + y sunt din [0, 1], atunci f (x + y) ≥ f (x) + f (y).
Demonstraţi că: f (x) ≤ 2x, ∀ x ∈ [0, 1].
Olimpiadă Irlanda
198. Să se determine a, b ∈ ℝ astfel încât funcţia f: [ 0,3] → ℝ,
 x2 x ∈ [ 0,1)

f(x) =  2 x 2
x ∈ [1, 2] să aibă proprietatea că orice y ∈ [0,8] este
ax + b x ∈ ( 2,3]

imaginea
prin f a unei singure valori x ∈ [ 0,3] .
Ion Cuculescu, concurs 1976

199. Determinaţi funcţiile f: (1, ∞ ) → ℝ care satisfac relaţia


f (xyz) = x f (y) + y f (z) + +z f (x), ∀ x, y, z > 1.
Dorel Miheţ, concurs G.Moisil 1997
200. Să se arate că nu există funcţii strict monotone f: ℝ → ℝ cu proprietatea:
f (a + b – x) = f (a – x) + f (x – b), ∀ x ∈ ℝ, unde a, b ∈ ℝ sunt fixate.
Lucian Dragomir, OJ Caraş-Severin 1994
201. Determinaţi funcţiile f: ℕ → ℕ pentru care avem
f (m 2 + f (n)) = f 2 (m) + n, ∀ m, n ∈ ℕ.
Lucian Dragomir, OJ 2001
202. Găsiţi funcţiile f: ℝ → ℝ cu proprietatea:f (x) + f ( [ x ] ) + f ( { x} ) = 2x,∀ x ∈ ℝ.
Dorel Miheţ, RMT 2000
203. Fie F o mulţime fixată cu n elemente. Determinaţi câte mulţimi E ⊂ ℝ au
proprietatea următoare: pentru f: E → F, f (x) = x 2 avem f (E) = F.
Adrian Ghioca, OJ 1985
204. Fie E o mulţime finită şi f: E → E o funcţie cu proprietatea că
( f f )( x ) = x, ∀x ∈ E. Demonstraţi că dacă E are un număr impar de elemente, atunci
există k ∈ E astfel încât f (k ) = k
Gh. Ionescu, OJ 1977
205. Fie f: ℕ → N o funcţie crescătoare cu proprietatea că există x ∈ ℕ astfel încât f(x)
< x.

49
Să se arate că există y ∈ ℕ astfel încât f (y) = y.
Marius Gârjoabă, OJ Sibiu 1994
206. Să se determine n ∈ ℕ * astfel încât x − 1 + x − 2 + ... + x − n < x , ∀ x ≥ n.
Laurenţiu Panaitopol, concurs GM 1996

207. Fie f : ( 0, ∞ ) → ( 0, ∞ ) o funcţie cu proprietatea că ( f f ) ( x) = x 2 , ∀x ∈ ( 0, ∞ )


Arătaţi că : a) f este bijectivă :
b) f ( x) = f ( x ), ∀x ∈ ( 0, ∞ ) .
Pal Dalyay , OJ 1982
 ax + b , x ≤ 1
208. Se consideră funcţiile f , g : ℝ → ℝ, f ( x) =  , g ( x) = 2 x − 1. Să se arate
cx + d , x > 1
că o condiţie necesară şi suficientă pentru ca să existe h : ℝ → ℝ aşa încât
h f = f h = g este a + b = c + d = 1 şi ac > 0.
I.V.Maftei , M.Piticari , OJ 1984
209. Fie A = B ∪ C şi funcţia f : P( A) → P ( A) × P ( A), f ( X ) = ( X ∪ B, X ∪ C ) .
Demonstraţi că f este injectivă dacă şi numai dacă B ∩ C = ∅.
Dorel Miheţ , OJ 1984
210. a) Dacă M ⊂ ℝ este o mulţime finită,determinaţi funcţia strict crescătoare
f :M → M;
b) Dacă M ⊂ ℝ +* este o mulţime finită,determinaţi toate funcţiile f : M → M care
satisfac xf ( y ) = yf ( x), ∀x, y ∈ M .
I.Nedelcu , ON 1985
 0 ,n = 0
211. Demonstraţi că funcţia f : ℕ → ℝ, f (n) = 
( )
este
n 2n + 1 − 2n − 1 , n > 0
injectivă.
Mircea Becheanu , ON 1987
212. Fie a ∈ ℝ* . Arătaţi că există funcţii bijective f : ℝ → ℝ astfel încât

( )  ( )
a 2 f 2  a 2 + 1 x  − 2af x 2 + a 2 + 1 ≤ 0, ∀x ∈ ℝ dacă şi numai dacă a ∈ {−1,1}.
Ioan Tomescu , OJ 1988
213. Arătaţi că nu există funcţii f : ℤ → ℤ astfel încât ( f f )( x) = x + 1, ∀x ∈ ℤ.
C.Năstăsescu,S.Dăscălescu, ON 1988
π π  x 1
214. Arătaţi că funcţia f :  ,  →  3, 2  , f ( x) = tg + este bijectivă.
3 2 2 sin x
Liviu Pîrşan , OJ 1971
215. Arătaţi că dacă o funcţie este inversabilă şi impară, atunci inversa ei este impară.
OJ 1976

50
216. Se consideră funcţiile f , g , h : ℕ → ℕ cu proprietăţile : g şi h sunt bijective şi
f (n) = g (n) − h(n), ∀n ∈ ℕ. Demonstraţi că f (n) = 0, ∀n ∈ ℕ.
ON 1979

217. Fie n ∈ ℕ, n ≥ 3 un număr impar şi A = { x1 , x2 ,..., xn } .Determinaţi toate funcţiile


f : A → A care au proprietatea că : f ( x1 ) − x1 = f ( x2 ) − x2 = ... = f ( xn ) − xn
Romeo Ilie , ON 1992
x+m
218. Se consideră funcţia f : ℝ → [ a, b ] , f ( x) = 2 , a, b ∈ ℚ, m ∈ ℤ. Determinaţi
x + x +1
a, b, m astfel încât f să fie surjectivă.
Marius Burtea , ON 1993
219. Fie F = { f : ℝ → ℝ / f ( f ( x) + y ) = f ( x) + f ( y ), ∀x, y ∈ ℝ}.
a) Să se determine toate funcţiile injective din F;
b) Să se determine toate funcţiile surjective din F;
c) Să se arate că F conţine funcţii neconstante care nu sunt nici
injective,nici surjective.
Dorel Miheţ , ON 1993
220. Fie funcţia f : ℕ* → ℕ* definită prin f (n) = numărul de pătrate existente ăn
intervalul  n 2 ,2n 2  . Arătaţi că f este monoton crescătoare şi surjectivă.
 
Marius Burtea , ON 1995
221. Se consideră mulţimea M = {1, 2,..., n}. Fie A, B, C submulţimi ale lui M;
determinaţi numărul tripletelor (A, B, C) care satisfac simultan proprietăţile:
a) A ∪ B ∪ C = M ;
b) card ( A ∩ B ∩ C ) = 1.
Dorel Miheţ, OJ Bacău 2000
222. Determinaţi numărul submulţimilor mulţimii A = {1, 2,..., 2n} în care ecuaţia
x + y = 2n + 1 nu are soluţii.
Olimpiadă Polonia
223. Câte cuvinte distincte, de lungime 1997 pot fi formate utilizând literele A, B, C şi
numai acestea, fiecare de un număr impar de ori ?
concurs Ungaria, Israel 1997
224. Determinaţi numărul numerelor naturale n cu următoarele proprietăţi:
a) n are 1000 de cifre; b) toate cifrele lui n sunt impare;
c) modulul diferenţei oricăror două cifre vecine este 2.
concurs Irlanda
225. Să se determine numărul de elemente ale mulţimii
A = {N = a1a2 ...an / a i ∈ {2, 3, 7, 9}, 3 / N}, unde n ∈ ℕ, n ≥ 1 este un număr natural
nenul
fixat.

51
Dorel Miheţ, concurs 2003
226. Aflaţi câte numere naturale scrise în baza zece îndeplinesc simultan următoarele
condiţii:
(i) fiecare număr are 6 cifre;
(ii) suma cifrelor fiecărui număr este 9;

(iii) 4 dintre cifrele fiecărui număr sunt 2, 0, 0, 4.


Lucian Dragomir, OJ 2004
227. Fie A o mulţime cu n elemente (n > 1). Determinaţi numărul tripletelor ordonate de
mulţimi (A1, A2, A3) care satisfac simultan condiţiile:
(1) Ai ≠ ∅, 1 ≤ i ≤ 3; (2) A1 ∪ A2 = A2 ∪ A3 = A3 ∪ A1 = A;
(3) A1 ∩ A2 ∩ A3 = ∅.
M. Balaj, concurs G.Moisil 1996
228. Într-o sală sunt n matematicieni; fiecare dintre ei cunoaşte exact k matematicieni,
Care este valoarea minimă a lui k pentru a fi siguri că există cel puţin trei matematicieni
astfel încât fiecare să-i cunoască pe ceilalţi doi ?
Short list, OBMJ 2001

229. La un turneu de tenis au participat de două ori mai mulţi băieţi decât fete. Fiecare
pereche de participanţi a jucat exact o dată (şi nu au fost rezultate egale). Raportul între
7
numărul victoriilor obţinute de fete faţă de cele obţinute de băieţi a fost . Câţi
5
participanţi au fost la acest turneu ?
OBJ, 2000
( )
1981
230. Demonstraţi că în scrierea numărului 2501 + 50 ca număr zecimal primele
3962 cifre după virgulă sunt zerouri.
D.M.Bătineţu – Giurgiu, ON 1981
231. Un elev are 10 bile numerotate cu numerele 1, 2, 3,.., 10 şi trebuie să le pună în trei
urne identice astfel încât în nici o urnă să nu fie două bile numerotate cu numere
consecutive. În câte moduri se poate face aceasta ?
juriu ON 2002, clasa a VII a
232. Câte numere de n cifre, formate numai cu cifrele 1, 9, 8, 6 se divid cu 3 ?
Dorel Miheţ
233. Să se determine numărul submulţimilor nevide ale mulţimii {1, 2,..., 10} care nu
conţin numere consecutive.
***
234. Fie F o mulţime fixată cu n elemente. Determinaţi câte mulţimi E ⊂ ℝ au
proprietatea următoare: pentru f: E → F, f (x) = x 2 avem f (E) = F.
Adrian Ghioca, OJ 1985

52
235. Fie Pm familia de parabole de ecuaţii y = x 2 – mx – 1, unde m este un parametru real.
Notăm Pm ∩ Ox = {Am, Bm} şi Pm ∩ Oy = {Cm}. Determinaţi locul geometric al centrului
cercului circumscris triunghiului AmB mC m.
Dorel Miheţ, OJ Timiş, 1991
236. Fie x1 , x2 ,..., xn , n ≥ 2, numere reale pozitive astfel încât x1 + x2 + ... + xn = 1. Arătaţi

1 1 1
(1 + x1 )(1 + x2 ) ⋅ ... ⋅ (1 + xn ) < 1 + + + ... +
1! 2! n!
***

237. Determinaţi numărul funcţiilor f : {1, 2,..., n} → {0,1} care îndeplinesc condiţia :
a = f (1) + f (2) + ... + f (n) este un număr par.
***
238. Demonstraţi că pentru orice număr natural nenul n , există un număr natural k astfel
încât : Ck2 ≤ n < Ck2+1.
***
239. Se consideră mulţimea X = {1,2,..., n} , n ∈ ℕ, n ≥ 2. Numim partiţie a lui X un triplet
( A, B, C ) de submulţimi ale lui X cu proprietatea că
A ∪ B ∪ C = X , A ∩ B = B ∩ C = C ∩ A = ∅ (două triplete care diferă doar prin ordinea
termenilor se consideră identice).
a) În câte partiţii mulţimea care îl conţine pe 1 are k elemente ?
3n−1 + 1
b) Arătaţi că X are partiţii distincte.
2
Gheorghe Eckstein , OJ Timiş 1991
240. Dacă M este o mulţime finită cu n elemente,determinaţi numărul elementelor
fiecăreia dintre mulţimile A = {( X , Y ) / X , Y ⊂ M , X ∪ Y = M } şi
B = {( X , Y , Z ) / X , Y , Z ⊂ M , X ∪ Y ∪ Z = M } .
Gheorghe Andrei , OJ Bacău 1993
241. Arătaţi că pentru orice n ∈ ℕ numărul
*
C2nn este divizibil prin 2n − 1.
OJ Arad 1993
79
242. Găsiţi ultimele două cifre ale numărului 79 .
OJ Timiş 1993
243. Determinaţi numerele naturale nenule k şi n pentru care Cnk = n + k.
OJ Buzău 1996
244. Fie a = 4k − 1, k ∈ ℕ . Demonstraţi că suma
*
Sn = 1 − Cn2 a + Cn4 a 2 − Cn6 a 3 + Cn8 a 4 − ...
se divide cu 2n−1 , ∀n ∈ ℕ*.
Dorel Miheţ , OJ 1984

53
245. Se consideră un şir x0 , x1 ,..., xn ,... de numere reale care satisface condiţiile :
n
x0 = x1 = 1, ∑ xk ⋅ xn− k = 2n ⋅ xn , ∀n ∈ ℕ, n ≥ 2.
k =0
a) Determinaţi termenul general al şirului;
 n 
b) Arătaţi că mulţimea  yn = ∑ xk / n ∈ ℕ  este mărginită;
 k =0 
n
c) Calculaţi Sn = ∑ (k − 1) ⋅ xk .
k =2
OJ 1989
246. Determinaţi cel mai mic număr natural a pentru care < a , ∀n ∈ ℕ .
C2nn n *

Laurenţiu Panaitopol , OJ 1990


247. Determinaţi numărul şirurilor finite a1 , a2 ,..., an , unde
n
ak ∈ {0,1} ,0 ≤ ak + ak +1 ≤ 1, ∀k = 1, n − 1 şi ∑ ak = m.
k =1
Ioan Tomescu , ON 1990
248. Fie n ∈ ℕ , m ∈ ℕ, m ≤ n, σ m = Cn0 + C1n + ... + Cnm . Arătaţi
*
că :
σ 0 ⋅ Cn0 + σ1 ⋅ Cn1 + ... + σ n ⋅ Cnn = 22n−1 + C2nn−−11.
Florian Dumitrel , ON 1993

249. Determinaţi numărul perechilor ( X , Y ) care satisfac X ≠ ∅, X ⊂ Y ⊂ {1, 2,3,..., n}.


OJ Brăila 1998
250. Determinaţi numerele naturale x şi z , prime între ele , care satisfac
C2yx = C xy + y + 1.
Romeo Ilie , OJ Braşov 2000
n n −1
251. Demonstraţi că numerele a = ∑ 2k ⋅ C22nk şi b = ∑ 2k ⋅ C22nk +1 sunt prime între ele.
k =0 k =0
OJ Caraş-Severin 2000
252. Se consideră numerele
a = Cn0 + Cn3 + Cn6 + ... , b = C1n + Cn4 + Cn7 + ... , c = Cn2 + Cn5 + Cn8 + ...
Arătaţi că : a 2 + b 2 + c 2 − 3abc = 2n , ∀n ∈ ℕ* .
OJ Iaşi , 2001
253. Arătaţi că dacă ( an )n≥1 este un şir de numere reale nenule astfel încât
n
∑ ak ⋅ Cnk = an ⋅ 2n−1, ∀n ∈ ℕ* , atunci ( an )n≥1 este o progresie aritmetică.
k =1
Lucian Dragomir , OJ 2001
54
254. Pentru fiecare n ∈ ℕ, n ≥ 2 notăm cu f(n) numărul minim de elemente ale unei
mulţimi S care îndeplineşte simultan condiţiile :
(i) {1, n} ⊂ S ⊂ {1,2,..., n} ;
(ii) orice element din S diferit de 1 este egal cu suma a două elemente (nu
neapărat distincte) din S .
Demonstraţi că :
(a) f (n) ≥ 1 + [ log 2 n ] ;
(b) există o mulţime infinită de valori ale lui n pentru care f (n) = f (n + 1).
Dorel Miheţ , OJ 2002
255. Într-o urnă sunt 8 bile albe şi 24 de bile negre.Se scoate o bilă,apoi după ce se
introduce bila scooasă în urnă,se mai scoate o bilă,iar după ce s-a introdus şi
aceată bilă în urnă,se mai scoate o bilă.Calculaţi probabilităţile :
a) de a ieşi 3 bile albe:
b) de a ieşi 2 bile albe;
c) de a ieşi o bilă albă;
d) de a nu ieşi nici o bilă albă.
C.Ionescu-Ţiu,Gazeta Matematică

256. Într-o urnă sunt 8 bile albe şi 2 bile negre.Se extrag n bile.Determinaţi valorile lui
n astfel încât probabilitatea ca între bilele extrase să fie cel puţin o bilă neagră să
8
fie mai mare ca .
15
Lucia Ţene , OJ 1972

257. Un zar este construit astfel : o faţă este marcată cu un punct,două feţe cu câte două
puncte şi trei feţe cu câte trei puncte,deci o aruncare cu zarul poate conduce la
rezultatele 1 , 2 sau 3. Zarul se aruncă de două ori şi se notează rezultatele
obţinute.Care este probabilitatea ca:
a) cele două rezultate să fie egale ?
b) suma celor două rezultate să fie 4 ?
ON 1970
258. Într-o urnă sunt n bile numerotate cu 1 , 2 , ... , n . Din această urnă se extrage o
bilă.Care este probabilitatea ca numărul obţinut să fie un cub perfect sau un
multiplu de 9 micşorat cu 2 ?
OJ 1975
259. Se consideră n bile numerotate de la 1 la n. Se introduc la întâmplare în trei
urne.Care este probabilitatea ca într-o urnă să fie trei bile pe care sunt înscrise trei
numere multipli de trei ?
Marcel Chiriţă , OJ 1980
260. Să se determine probabilitatea ca dintr-o şcoală cu 1096 de elevi să existe cel puţin
4 elevi care să-şi serbeze ziua de naştere în aceeaşi zi a anului (se va considera
anul cu 365 de zile).
Ioan Tomescu , OJ 1974
55
261. Se consideră toate numerele de 6 cifre care se formează cu ajutorul cifrelor
1, 2,3, 4,5,6. Să se afle probabilitatea ca la o alegere a unui număr oarecare să
obţinem un număr care să nu înceapă cu 13.
OL Bucureşti 1976
262. Se consideră experienţa aruncării a două zaruri (unul roşu şi unul verde) şi fie x ,y
numerele ce apar pe feţele zarurilor la o aruncare, x, y ∈ {1, 2,3, 4,5,6} , x pe zarul
roşu şi y pe zarul verde.Notăm cu A( x, y ) punctul de coordonate x , y . Care este
probabilitatea ca la repetarea de 3 ori a experienţei, punctele A,B,C astfel obţinute
să fie colineare ?
Romanţa şi Ioan Ghiţă,Gazeta Matematică
n
263. Demonstraţi identitatea ∑ 3k +l ⋅ C3nn−k −l ⋅ C2nn−−kk −l = 33n , ∀n ∈ ℕ*.
k ,l = 0
Andras Szilard , Gazeta Matematică
264. Dacă un coleg vă spune: Pariez cu tine că cel puţin doi dintre primii cinci elevi
care vor răspunde azi la matematică vor lua aceeaşi notă , acceptaţi pariul ? Dar
dacă notele ce se acordă sunt doar 4 , 5 , ... , sau 10 ?
Nicolae Negoescu , RMT 2000
265. O dreaptă variabilă d trece prin punctul M (8, 2) şi face cu semiaxa (Ox un unghi
α > 900. Fie A şi B punctele de intersecţie ale dreptei d cu dreapta d1 : 2 x − y = 0 ,
respectiv cu axa Ox. Determinaţi coordonatele punctului A în cazul în care aria
triunghiului AOB este minimă.
Traian Covaciu , RMT

266. În reperul xOy se consideră punctele A(1, 2), B (3, 4). Determinaţi poziţia punctului
D ∈ Oy pentru care suma AD 2 + BD 2 este minimă.
Damian Marinescu , RMT 2005
267. Într-un plan raportat la un sistem de coordonate carteziene xOy se consideră
dreapta d : y = mx. Arătaţi că există o infinitate de puncte M având ambele
coordonate întregi şi care nu sunt situate pe dreapta d astfel încât simetricele lor N
faţă de dreapta d au de asemenea ambele coordonate întregi dacă şi numai dacă
m ∈ ℚ.
***
268. În reperul xOy se consideră mijloacele M (a, b), N (b, c), P (a, c) ale laturilor unui
triunghi.Dacă a,b,c sunt numere întregi, arătaţi că aria triunghiului este un număr
par.
Lucian Dragomir,OL Caraş-Severin 2004
269. Determinaţi vârfurile B şi C ale triunghiului ABC ştiind că A(2, 0) şi că dreptele
de ecuaţii d1 : y = x + 2, d 2 : x = 0 sunt înălţimi.
Lucian Dragomir,OL Caraş-Severin 2001

56
270. Într-un sistem cartezian de coordonate se consideră punctele M ( x, y ) ale căror
coordonate satisfac y lg x + x 2lg y = 2. Arătaţi că mulţimea acestor puncte reprezintă
două semidrepte perpendiculare.
Ol Cluj 2002,OL Constanţa 2004
271. Se dă familia de drepte d m : m 2 x + 2(m + 1) y − (m + 2)2 = 0, m ∈ ℝ. Determinaţi
regiunea din plan acoperită de dreptele acestei familii.
Laurenţiu Panaitopol,OJ 1972
272. Se dau patru drepte concurente dintre care două sunt perpendiculare. O dreaptă
variabilă le intersectează pe acestea din urmă în A şi N şi pe celelalte două în M şi
B.
MA NB
Arătaţi că ⋅ este constant.
MB NA
OJ 1973
273. Un pătrat ABCD de latură variabilă are vârfurile A şi B pe axa Ox, respectiv pe Oy
şi centrul P de abscisă constantă. Ştiind că pătratul se află în cadranul întâi :
a) Demonstraţi că centrul pătratului este un punct fix;
b) Găsiţi locul geometric al punctului C şi locul lui D ;
c) Determinaţi lungimea minimă, respectiv maximă a laturii pătratului ABCD.
C.Ottescu , OJ 1974
274. Fie ABC un triunghi echilateral fixat şi d o dreaptă variabilă din planul
triunghiului.Fie M,N,P proiecţiile vârfurilor A,B,C pe dreapta d. Determinaţi
minimul sumei AM 2 + BN 2 + CP 2 în funcţie de lungimea laturii triunghiului şi
precizaţi mulţimea dreptelor din planul triunghiului pentru care se realizează acest
minim.
Marcel Chiriţă, OJ 1982
275. Se consideră în reperul xOy mulţimea
 (1 + t )2 , y = 1 + 6t , t ∈ ℝ* − 4 . Arătaţi că E este reuniunea a
E = ( x , y ) / x = { }
 4t − t 2 4t − t 2 
două semidrepte.
C.Udrişte , OJ 1983
276. Pe laturile triunghiului ABC se construiesc în exterior triunghiurile dreptunghice
isoscele ABD, cu AB = BD , respectiv ACE , cu AC = CE . Să se arate că
înălţimea din A a triunghiului ABC şi dreptele CD şi BE sunt concurente.
OJ 1984
277. Fie ABC un triunghi şi P un punct variabil în planul triunghiului.Se notează cu
D,E,F simetricele punctului P faţă de mijloacele laturilor (BC),(CA),(AB). Arătaţi
că dreptele AD,BE,CF sunt concurente într-un punct notat M , iar dreapta PM trece
printr-un punct fix care se cere precizat.
OJ 1989

57
278. În reperul cartezian xOy se consideră punctul A(4,0) şi dreptele de ecuaţii
d1 : 2 x + y + 2 = 0, d 2 : 3 x − y − 2 = 0. Să se găsească punctele B ∈ d1 , C ∈ d 2 astfel
încât d1 să fie înălţime , iar d 2 mediană în triunghiul ABC.
Lucian Dragomir , OL Caraş-Severin 2002
AM
279. Fie ABC un triunghi dreptunghic în A şi M ∈ ( AB) astfel încât = 3 3 − 4. Să
MB
se determine măsura unghiului B ştiind că simetricul lui M faţă de mijlocul
segmentului (GI) aparţine dreptei AC. (G este centrul de greutate al triunghiului ,
iar I este centrul cercului înscris).
Marian Andronache , ON 2002
280. În reperul cartezian xOy se consideră dreptele de ecuaţii
d1 : 2 x − y − 2 = 0, d 2 : x + y − 4 = 0, d3 : y = 2, d 4 : x − 4 y + 3 = 0. Să se găsească
vârfurile triunghiurilor care au medianele d1 , d 2 , d3 şi în care d 4 este una dintre
înălţimi.
Lucian Dragomir , OJ 2001

281. Fie patrulaterul ABCD şi numerele reale pozitive subunitare m, n. Pe laturile AB,
BC, CD, DA se consideră puncte E, F, G, H încât AE = mAB, BF = nBC, DG =
mDC, AH = nAD. Fie I intersecţia lui EG cu FH. Să se calculeze rapoartele
IE IF
, .
IG IH
Concursul Radu Miron
282. Fie ∆ ABC un triunghi ascuţitunghic cu vârfurile de afixe z A , z B , zC şi
| z A |=| z B |=| zC |= R . Să se demonstreze că:
1 1 1 3
+ + ≥ .
| z A + z B | | z B + zC | | zC + z A | R
Concursul Radu Miron
283. Se consideră un triunghi ABC şi punctele M ∈ ( BC ) , N ∈ ( CA ) , P ∈ ( AB ) astfel
AP BM CN
încât = = . Arătaţi că, dacă triunghiul MNP este echilateral, atunci
PB MC NA
triunghiul ABC este echilateral.
OJ 2006
( )
284. Rezolvaţi în mulţimea numerelor reale ecuaţia x + log 3 x + x + 1 = 2 x + log 3 x.
2 2

Lucian Dragomir, Concurs RMCS, 2006 ( ediţia I )

285. Se consideră trei numere complexe u , v, w de modul 1. Arătaţi că există o alegere a


semnelor + şi − astfel încât ±u ± v ± w ≤ 1.
Dan Schwarz, OJ 2007

58
286. Demonstraţi că ecuaţia z n + z + 1 = 0 are o soluţie complexă de modul 1 dacă şi
numai dacă restul împărţirii lui n la 3 este 2.
ON 2007, Piteşti
+x
+ log 2 x = 2 x +1.
2
287. Rezolvaţi ecuaţia 2 x
Lucian Dragomir, ON 2007, Piteşti
288. Studiaţi dacă există funcţii bijective f : ℝ → ℝ care au proprietatea că

( ) 1
f 3x − f 2 ( 2 x + 1) ≥ , ∀x ∈ ℝ.
4
Lucian Dragomir, Concurs RMCS, 2007
( )
289. Determinaţi numerele întregi x pentru care log3 1 + 2 x = log 2 (1 + x ) .
Lucian Dragomir, OJ 2008
290. Se consideră o funcţie f : ℝ → ℝ cu proprietatea
 x + y  f ( x) + f ( y )
f = , ∀x, y ∈ ℝ.
 3  2
a) Demonstraţi că funcţia g : ℝ → ℝ, g ( x) = f ( x) − f (0) este aditivă, adică
g ( x + y ) = g ( x) + g ( y ), pentru orice x, y ∈ ℝ.
b) Arătaţi că f este constantă.
Dorel Miheţ, OJ 2008
291. Se consideră triunghiul ABC şi punctele D ∈ ( BC ) , E ∈ ( CA ) , F ∈ ( AB ) astfel încât
BD CE AF
= = . Demonstraţi că, dacă centrele cercurilor circumscrise triunghiurilor
DC EA FB
DEF şi ABC coincid, atunci triunghiul ABC este echilateral.
Dana Heuberger, ON 2008, Timişoara
292. Demonstraţi că dacă numerele complexe a, b, c verifică egalitatea
a ⋅ bc + b ⋅ ca + c ⋅ ab = 0, atunci ( a − b )( b − c )( c − a ) ≥ 3 3 abc .
Bogdan Enescu, ON 2008, Timişoara
293. Determinaţi numerele naturale m şi n pentru care egalitatea
m n
x ⋅ x 2 ⋅ n x3 ⋅ m x = x 2 este adevărată pentru orice x > 0, x ≠ 1.
Lucian Dragomir, OL Teleorman 2009 ( RMCS )
294. Determinaţi funcţiile injective f : ℝ → ℝ cu proprietatea că
f ( x) ⋅ f ( y ) = x ⋅ f ( y ) + y ⋅ f ( x) − f ( xy ), ∀x, y ∈ ℝ.
Iacob Didraga, OL Caraş – Severin, 2009 (RMCS)
(
295. Arătaţi că nu există numere întregi x pentru care log3 ( x + 2) = log 2 2 x + 7 .)
Lucian Dragomir, Concurs RMCS 2010
296. Se consideră numerele complexe distincte şi nenule u şi w.

59
Demonstraţi că zw + w ≤ zv + v , pentru orice z ∈ ℂ, z = 1, dacă şi numai dacă
există
k ∈ [ −1,1] astfel încât w = k ⋅ v.
Dan Marinescu, ON 2010, Iaşi
297. Se consideră şirul ( an )n≥0 de numere reale strict pozitive, pentru care
n
∑ Cnk ⋅ ak ⋅ an−k = an2 , pentru orice n ∈ ℕ.
k =0
Arătaţi că şirul considerat este o progresie geometrică.
Lucian Dragomir, ON 2010, Iaşi
298. În exteriorul triunghiului neechilateral ABC se consideră triunghiurile asemenea
ABM, BCN şi CAP astfel încât triunghiul MNP să fie echilateral. Determinaţi măsurile
unghiurilor triunghiurilor ABM , BCN şi CAP.
ON 2010, Iaşi ( colectivul de bourbăcuţi din judeţul Hunedoara)
299. Arătaţi că, dacă z1 , z2 , z3 , z4 sunt numere complexe distincte, cu modulele egale şi
cu suma nulă, atunci patrulaterul cu vârfurile de afixe z1 , z2 , z3 , z4 este dreptunghi.
Aurel Bârsan, OJ 2011
300. Fie a, b două numere complexe. Arătaţi că afirmaţii sunt echivalente :
(1) Modulele soluţiilor complexe ale ecuaţiei x 2 − ax + b = 0 sunt respectiv egale cu
modulele soluţiilor ecuaţiei x 2 − bx + a = 0.
(2) a 3 = b3 sau b = a.
Mihai Bălună, OJ 2011

60

S-ar putea să vă placă și